Obstetrics and Gynaecology Flashcards

1
Q

A nervous 42yo woman presents herself to the antenatal clinic very worried that she has missed right time to have the combined test for Down’s syndrome screening. She is now 17 weeks pregnant and is v.concerned about her age. You counsel her about the appropriate alternative, the quadruple test and arrange to have this done. What assays make up the quadruple test?

A - AFP, PAPP-A, Inhibin B, beta hCG

B - Unconjugated oestradiol, hCG, AFP and Inhibin A

C - beta hCG, PAPP-A, Nuchal translucency, Inhibin A

D - AFP, Inhibin B, beta hCG, Oestradiol

E - Unconjugated oestradiol, PAPP-A, beta hCG, Inhibin A

A

B - Unconjugated oestradiol, hCG, AFP and Inhibin A

At 42 she is at a relatively high risk (1 in 55). The combined test is only reliable between 10-13 weeks, it utilises PAPP-A and beta-hCG and the Nuchal Translucency scan. After 13 weeks it is no longer an accurate test - This excludes answer A,C,D, and E. How I remember - The quadruple test has nothing to do with ‘B’ or beta. Its Inhibin A, Alpha Fetoprotein, and hCG (NOT beta hCG)

How well did you know this?
1
Not at all
2
3
4
5
Perfectly
2
Q

a 33yo nulliparous woman is 29wks pregnant. She was referred to the Rapid access clinic for investigation of a solitary breast lump. Unfortunately the biopsy showed a carcinoma. After much counselling a decision is made on further treatment. What options are available to her?

A - Tamoxifen

B - CT Abdo-Pelvis

C - Radiotherapy

D - Chemotherapy

E - Bone Isotope scan to look for mets

A

D - Chemotherapy

A difficult question to answer due to the lack of information on how aggressive the cancer is. But in general terms, Tamoxifen is contraindicated in pregnancy and breastfeeding as it is highly teratogenic CT and the Bone scan are both unacceptable levels of radiation for a non therapeutic intervention. Radiotherapy is a last resort in pregnancy Chemotherapy can be used in the second and third trimesters Regardless a course of Betamethasone should be started to aid lung development in anticipation of an early delivery.

How well did you know this?
1
Not at all
2
3
4
5
Perfectly
3
Q

A 38yo woman with DM (type 2) attends clinic. she has a BMI of 48 and is controlling her blood sugars with insulin. You have a long discussion with this woman about her weight. What should not be routinely offered to this woman?

A - Post-natal thromboprophylaxis

B - Vitamin C 10mg BD

C - Regular screening for pre-eclampsia

D - Referral to an obstetric anesthetist

E - An active 3rd stage of labor as increased risk of postpartum hemorrhage.

A

B - Vitamin C

It should be vitamin D

Obese women need to be offered weight-loss support, high dose folic acid and diabetic screening. VTE risk is high for the obese and for pregnancy Pre-eclampsia screening should be offered to all Obese women offer an increased challenge for anesthesia so should be referred there is also an increased risk of PPH, so an active 3rd stage is called for.

How well did you know this?
1
Not at all
2
3
4
5
Perfectly
4
Q

A nulliparous woman is seen at the antenatal clinic 27 weeks into her first pregnancy. Routine screening with a 75g oral glucose tolerance test for gestational DM is performed. which result would confirm a diagnosis of GDM?

A - Fasting plasma venous glucose of greater than 5.0 micromoles/L

B - 2-hour plasma venous glucose of greater than 7.8 micromoles/L

C - Random plasma venous glucose of greater than 4.8 micromoles/L

D - 2-hour plasma venous glucose of less than 7.0 micromoles/L

E - 2-hour plasma venous glucose of less than 7.8 micromoles/L

A

B - 2-hour plasma venous glucose of greater than 7.8 micromoles/ L

GDM is very common, affecting 2-5% of pregnancies in the UK. Risk factors include - previous macrosomic baby, previous GDM, high BMI, positive family history and ethnicity. Those at risk should be screened. The WHO defines GDM as encompassing impaired glucose tolerance and diabetes ( a fasting greater than 5.6 or a 2 hour greater than 7.8)

How well did you know this?
1
Not at all
2
3
4
5
Perfectly
5
Q

A 29yo attends her booking visit and has screening bloods taken. Which of these are the most appropriate tests?

A - Hepatitis C, HIV, Syphilis and Toxoplasmosis

B - Rubella, Hepatitis B, Hepatitis C, Sypilis

C - Syphilis, Rubella, Hep B, HIV

D - HIV, CMV, Rubella, Hep B

E - HIV, Syphilis, Rubella, and group B Streptococcus

A

C - Syphilis, Rubella, Hep B, HIV

The above is recommended by NICE Toxoplasmosis and CMV are too infrequent in the population to be warranted inclusion in a screening program. Hep C screening is not cost effective.

How well did you know this?
1
Not at all
2
3
4
5
Perfectly
6
Q

A 30yo nulliparous woman is 29wks. She presented to hospital with minor, painless, unprovoked PV bleeding of about a teaspoon full. Her anomaly scan at 20wks showed a low lying placenta. Her fetus is moving well and CTG is reassuring. What is the most appropriate management?

A - Allow home as it is a small bleed

B - Admit and give steroids

C - Admit, IV access, observe bleed free for 48hrs before discharge.

D - Admit, IV access, Group and save and administer steroids if there is further bleeding.

E - Group and save, FBC and allow home; review in clinic in a week.

A

D - Admit, IV access, Group and save and administer steroids if there is further bleeding.

Bleeding in pregnancy is very common. need to be aware of a placenta praevia or placental abruption. Abruptions tend to be large painful bleeds. The small bleed could precede a large bleed so discharge is the wrong answer. Steroids at this stage are not indicated 48hrs is a bit too long to keep the woman in hospital due to nosocomial risks.

How well did you know this?
1
Not at all
2
3
4
5
Perfectly
7
Q

A 34yo woman attends antenatal clinic for a routine ultrasound scan. Abnormalities of placentation are detected and an MRI organised. The MRI report shows; ‘The placenta is in the lower anterior uterine wall with evidence of invasion to the posterior wall of the bladder’. What is the most likely diagnosis?

A - Placenta Accreta

B - Placenta Percreta

C - Placenta Increta

D - Placenta Praevia

E - Ectopic Pregnancy

A

D - Placenta Praevia

Placenta Accreta is the firm adhesion of the placenta to the uterine wall, Increta is the invasion through the myometrium and percreta is invasion beyond the myometrium.

How well did you know this?
1
Not at all
2
3
4
5
Perfectly
8
Q

A 28yo pregnant woman attend A&E with a history of clear vaginal loss. She is 18 weeks pregnant and so far has had no problems. Her medical history includes a large cone biopsy of the cervix and an allergy to penicillin. She is worried as the fluid continues to come and now there is some blood. On examination it is apparent that her membranes have ruptured. What is the most appropriate initial management?

A - Discharge, Ultrasound scan the next day

B - Offer her a termination as its not possible for this pregnancy to continue.

C - Admit, Infection markers. Ultrasound and steroids

D - Ultrasound, infection markers and observation

E - Discharge and explain that she will probably miscarry at home.

A

D - Ultrasound, infection markers and observation

The outlook for this pregnancy is poor but there is a chance the pregnancy will continue, so option B is not correct.

The risk of chorioamnionitis precludes sending her home immediately, ruling out E and A

Before 24 weeks there is no role for steroids so option

How well did you know this?
1
Not at all
2
3
4
5
Perfectly
9
Q

A 37yo woman in her 4th ongoing pregnancy presents to the labour ward at 34 week’s gestation complaining of a sharp pain in her chest, worse on inspiration. An ABG shows: pH 7.51, PO2 8.0kPa, PCO2 4.61, base excess 0.9. What is the most appropriate investigation?

A - CTPA

B - MRI

C - D-dimer

D - Ventilation/perfusion Scintigraphy

E – Ultrasound

A

D - Ventilation/perfusion Scintigraphy

Ultrasound and MRI are not useful for confirming a PE

D-dimer is mainly useful as a predictor, it is also raised in pregnancy anyway

So it comes down to a V/Q scan or a CTPA. Both are diagnostically useful but a V/Q has by far the lower dose of ionizing radiation so is preferred in pregnancy.

How well did you know this?
1
Not at all
2
3
4
5
Perfectly
10
Q

A 32-year-old woman in her second pregnancy presents at 36 weeks gestation
with a history of passing a gush of blood stained fluid from the vagina an hour
ago, followed by a constant trickle since. The admitting obstetrician reviews her
history and weekly antenatal ultrasound scans have shown a placenta praevia.
What is the most appropriate management? She has a firm, posterior cervix and
has not been experiencing any contractions.

A. Induction of labour with a synthetic oxytocin drip
B. Cervical ripening with prostaglandins followed by a synthetic oxytocin drip
C. Digital examination to assess the position of the fetus
D. Monitor for 24 hours and manage as for preterm pre-labour rupture of
membranes (PPROM)
E. Caesarean delivery

A

E. Caesarean delivery

The low lying placenta in this case immediately precludes a vaginal delivery in any case, ruling out A and B.

The gush of blood and steady trickle implys rupture of membranes, premature in this case. Being 36 weeks makes the PPROM pathway rather pointless as it applies mainly to pre-34/36 weeks.

Option C is contraindicated outside of a pre-term labour scenario due to risk of infection

How well did you know this?
1
Not at all
2
3
4
5
Perfectly
11
Q

Maternal physiology changes throughout pregnancy to cope with the additional
demands of carrying a fetus. Which of the following changes best represents a
normal pregnancy?

A. Stroke volume increases by 10 per cent by the start of the third
trimester
B. Plasma volume increases disproportionately to the change in red cell
mass creating a relative anaemia
C. Plasma levels of fibrinogen fall, reaching a trough in the mid-trimester
D. Systemic arterial pressure rises to 10 mmHg above the baseline by term
E. Aortocaval compression reduces venous return to the heart, in turn
increasing pulmonary arterial pressure

A

B. Plasma volume increases disproportionately to the change in red cell
mass creating a relative anaemia

Stroke volume is over 30% higher at the start of the third trimester

There is an increase in fibrinogen and factors VII, X and XII

There is no change in the systemic of pulmonary blood pressure

How well did you know this?
1
Not at all
2
3
4
5
Perfectly
12
Q

A 30-year-old woman attends the antenatal clinic asking to be sterilized at the
time of her elective caesarean. She is 34 weeks into her second pregnancy having
had her first child 2 years ago via an emergency caesarean section. She is not sure
that she wants any more children. Further more, she does not wish to try for a
vaginal birth. She has tried the contraceptive pill in the past but does not like the
side effects. You talk to her about other options, including the sterilization she is
requesting. What is the best management option for this woman?

A. Mirena coil
B. Sterilization at the time of her caesarean section
C. T380 coil
D. Implanon
E. Vasectomy

A

C. T380 coil

If she is sure she doesn’t want a hormonal method then A and D (depot injection) are out.

You can’t suggest a vasectomy wiothout the partner in the consultation!

C (copper coil) is the best option as it can be reversed if needed and it is less of a risk than performing a sterilisation.

If she insists then a second opinion will be needed due to her young age.

How well did you know this?
1
Not at all
2
3
4
5
Perfectly
13
Q

A 41-year-old multipara attends the antenatal clinic at 36 weeks gestation
complaining of lower abdominal cramps and fatigue when mobilizing. Clinical
examination is unremarkable save for a grade I pansystolic murmur, loudest over
the fourth intercostal space in the midaxillary line. What is the most appropriate
management?

A. Urgent outpatient echocardiogram and referral to a maternal–fetal
medicine consultant
B. Reassurance and a 38-week antenatal clinic follow-up
C. Admission and work-up for cardiomyopathy
D. Post-natal referral to a cardiologist
E. Admission to the labour ward for induction of labour

A

B. Reassurance and a 38-week antenatal clinic follow-up

Dilation of the tricuspid valve leading to a mild regurgitant murmur is a normal consequence of pregnancy.

At this gestation abdominal pain and fatigue are also normal.

Any cardiac investigations are likely to cause alarm for no reason. Equally induction is not indicated in the 38th week for no cause.

How well did you know this?
1
Not at all
2
3
4
5
Perfectly
14
Q

A 32-year-old HIV positive woman who booked for antenatal care at 28
weeks gestation arrives on the delivery suite at 37 weeks with painful regular
contractions and a cervix dilated to 4 cm. Ultrasonography confirms a breech
singleton pregnancy with a reactive fetal heart rate. What is the most appropriate
management option?

A. Await onset of labour, avoid operative delivery, wash the baby at
delivery
B. Induce labour with synthetic prostaglandins
C. Await onset of labour, but have a low threshold for expediting vaginal
delivery using forceps
D. Await onset of labour, avoid operative delivery, administer steroids to
the infant immediately after birth
E. Caesarean delivery, wash the baby at delivery

A

E. Caesarean delivery, wash the baby at delivery

In HIV there is a requiremtn to avoid instrument delivery and amniocentisis. (C)

Generally a caeserean should be performed as it reduces the risk of vertical transmission. (A) is not perfered for that reason

Steroids have no place here at all (D)

There is also no benifit to expidiating a vaginal delivery for the above (B)

How well did you know this?
1
Not at all
2
3
4
5
Perfectly
15
Q

A 41-year-old multiparous woman attends accident and emergency at 32 weeks
gestation complaining of sudden onset shortness of breath. A CTPA demonstrates
a large saddle embolus. What is the most appropriate treatment regimen?

A. Load with warfarin to achieve a target international normalized ratio
(INR) of 3.0
B. Load with warfarin to achieve a target international normalized ratio
(INR) of 2.5
C. Load with warfarin to achieve a target international normalized ratio
(INR) of 20
D. 80 mg enoxaparin twice daily
E. 7.5 mg fondaparinux once daily

A

D. 80 mg enoxaparin twice daily

Warfarin is teratogenic so thats 3 options out

Of the two LMWH, both are efficous in treating pulmonary embolism, butonly Enoxaparin is licenced in pregnancy.

How well did you know this?
1
Not at all
2
3
4
5
Perfectly
16
Q

A 21-year-old woman attends the labour ward with per vaginal bleeding of 100 mL.
She is 32 weeks pregnant and has had one normal delivery in the past. An important
history to note is that of an antepartum haemorrhage in her last pregnancy and she
smokes 10 cigarettes a day. Her 20-week anomaly ultrasound revealed a posterior
fundal placenta. She admits she and her partner had intercourse last night and is
concerned by terrible abdominal pains. What is the most likely diagnosis?

A. Vasa praevia
B. Placenta praevia
C. Placenta accreta

D. Placental abruption
E. Cervical ectropion

A

D. Placental abruption

Her anomoly scan rules out (B)

(A) is a rare complication at the time of rupture of membranes that can lead to fetal demise, it is typically painless

(C) is diagnosed at the time of placental delivery

(E) would be a possibility, especially as it is common in pregnancy and intercourse can lead to bleeding. The severe pain goes strongly against this and leads to a likely diagnosis of abruption.

How well did you know this?
1
Not at all
2
3
4
5
Perfectly
17
Q

At a booking visit a first time mother is told that she is rhesus negative. Which of
these answers is the most appropriate advice for the mother?

A. It is important to have anti-D as it will make sure your baby does not
develop antibodies
B. If you have any bleeding before 12 weeks be sure to get an injection of
anti-D
C. Anti-D will stop your body creating antibodies to your baby’s blood
that may help protect the health of your next child
D. If your partner is rhesus negative you do not need to have anti-D
E. You need one injection that will cover your pregnancy even if you have
episodes of vaginal bleeding

A

C. Anti-D will stop your body creating antibodies to your baby’s blood
that may help protect the health of your next child

Having anti-D has no efect on a first pregnancy, it is also not nessesary before 12 weeks. Thats A and B excluded.

Although D is true, the risk of the partner not being the father excludes this (according to the book anyway)

And in the case of bleeding you will need further doses, excluding E

How well did you know this?
1
Not at all
2
3
4
5
Perfectly
18
Q

A 42-year-old para 4 with a dichorionic–diamniotic (DCDA) twin pregnancy at 31 weeks gestation presents to hospital with a painful per vaginam bleed of 400 mL. The bleeding seems to be slowing. She is cardiovascularly stable, although having abdominal pains every 10 minutes. There is still a small active bleed on speculum and the cervix appears closed. Both fetuses have reactive CTGs. She has had no problems antenatally and her 28-week ultrasound revealed both placentas to be well away from the cervix. What is your preferred management plan?

A. Admit to antenatal ward, ABC, iv access, Group and Save, CTG,
steroids, consider expediting delivery
B. Reassure and ask to come back to clinic next week if there are any problems
C. Admit for observation, iv access
D. Admit to labour ward, ABC, iv access, full blood count, cross-match 4
units of blood, CTG, steroids, consider expediting delivery
E. As bleeding settled and placenta not low, offer admission but arrange
follow-up if refused

A

D. Admit to labour ward, ABC, iv access, full blood count, cross-match 4
units of blood, CTG, steroids, consider expediting delivery

The significant PV bleed is concerning for a placental abruption. This is a sequale more common in twin pregnancies.

B and C both fail to take into account the seriousness of the situation. The woman should not be allowed to go home for the same reason (E).

Between A and D, the differnce is that in D you are more worried about blood loss so are getting crossmatched units, an appropriate step in this case.

How well did you know this?
1
Not at all
2
3
4
5
Perfectly
19
Q

You are the FY1 covering the antenatal ward. A 27-year-old nulliparous woman who is 36 + 5 pregnant has been admitted to your ward with
suspected pre-eclampsia. The emergency buzzer goes off in her room. You are the first to attend and find your patient flat on the bed having a generalized seizure – what do you do?

A. Call for help, ABC, nasopharyngeal airway, iv access and wait for fit to stop
B. Call for help, ABC, protect her airway, prepare for grade 1 caesarean section
C. Call for help, ABC, left lateral tilt, wait for seizure to end, listen in to fetus
D. Call for help, ABC, left lateral tilt, protect airway, prepare magnesium
E. Call for help, ABC, protect airway, prepare magnesium, check blood pressure

A

D. Call for help, ABC, left lateral tilt, protect airway, prepare magnesium

The first steps in this suspected eclamptic fit are to lie her flat and tilting to the left lateral (to prevent obsrtruction of venous return and in case of vomiting), and calling for help.

The options of C and D are best in line with this management, in addition you do need to protect her airway and prepare Mg to stabilise. This leaves otion D.

Of the other two options, inserting an airway adjunct or a cannula in a fitting patinet is a risk and is not an immediate management.

B may actually be the reality, but in this question you are an F1 and need to be managing the patient not preparing for surgery.

How well did you know this?
1
Not at all
2
3
4
5
Perfectly
20
Q

A 38-year-old woman in her first pregnancy is 36 weeks pregnant. She presents to the labour ward feeling dizzy with a mild headache and flashing lights. Her past medical history includes systemic lupus erythematosus (SLE), renal stones and malaria. Her blood pressure is 158/99 mmHg with 2+ protein in her urine. Her platelets are 55 × 109/L, Hb 10.1 g/dL, bilirubin 62 μmol/L, ALT 359 IU/L, urea 2.3 mmol/L and creatinine 64 μmol/L. What is the most likely diagnosis?

A. Thrombotic thrombocytopenic purpura (TTP)
B. HELLP syndrome
C. Idiopathic thrombocytopenic purpura (ITP)
D. Systemic lupus erythematosus (SLE)
E. HIV

A

B. HELLP syndrome

TTP is charecterised by; micoangiopathic haemolytic anemia, hrombocytopenia, fever, neurological involvement and renal impairment. In this case renal function is normal.

ITP is a diagnosis of exclusion

there isn’t anything to sugesst this is caused by HIV or SLE

This woman has haemolysis, elevated liver enzymes, and low platelets, therefore HELLP

How well did you know this?
1
Not at all
2
3
4
5
Perfectly
21
Q

A 19-year-old woman in her first pregnancy presents to the GUM clinic with
an outbreak of primary herpes simplex infection on her labia. She is 33 weeks
pregnant. What is the best advice regarding her herpes?

A. Aciclovir from 36 weeks until delivery
B. Caesarean section should be performed if she labours within the next 8
weeks

C. Reassure as the infection will pass and pose no further concern
D. If she labours within 6 weeks, a caesarean should be recommended
E. Aciclovir for 10 days and an elective caesarean at 39 weeks

A

D. If she labours within 6 weeks, a caesarean should be recommended

There is no evidence that antenatal Aciclovir reduces vertical transmission (A and E)

it takes 6 weeks until the infection is considered to be clear, if she labours within that period a caesarean should be reccomended. If she refuses then per-natal IV aciclovir has a role.

Unconditional reassurance (C) is not appropriate due to the above.

How well did you know this?
1
Not at all
2
3
4
5
Perfectly
22
Q

A 33-year-old woman presents to hospital with a 2-day history of itching
on the soles of her feet and the palms of her hands. Her pregnancy has been
straightforward and she has good fetal movements. Liver function tests reveal an alanine transaminase (ALT) of 64 IU/L and bile acids of 30 μmol/L. You suspect that she might have developed obstetric cholestasis. Which of the following bits of advice is true?

A. She could have intermittent monitoring in labour
B. Ultrasound and CTG surveillance help prevent stillbirth

C. Poor outcomes can be predicted by bile acid levels
D. Ursodeoxycholic acid (UDCA) helps prevent stillbirth
E. Meconium stained liquor is more common in labour

A

E. Meconium stained liquor is more common in labour

the itching and deranged LFTs (especially the bile acid) is typical of obstetric cholestasis. The liver function should be checked weekly. Stillbirth is a risk, so induction in week 37-38 is reccomended. Meconium stained liquor is more common.

UDCA treats the symptoms of OC but has no effect on stillbirth rates. (D) there is no link between outcomes and bile acid levels (C) and constant CTG monitoring in Labou is warrented (A).

option B is just ilogical

How well did you know this?
1
Not at all
2
3
4
5
Perfectly
23
Q

A 24-year-old woman who is 32 weeks pregnant presents to the labour ward with a terrible headache that has not improved despite analgesia. It started 2 days ago and came on suddenly. She has stayed in bed as it hurts to be in sunlight and she vomited twice this morning. Her past medical history includes a macroprolactinoma (which has been removed) and occasional migraines. She is haemodynamically stable with no focal neurology or papilloedema. You arrange for her to have a CT of her head as an emergency, which adds no further information to aid your
diagnosis. There are red cells on lumbar puncture but no organisms are isolated. What is the most likely diagnosis?

A. Migraine
B. Viral meningitis
C. Cerebral vein thrombosis (CVT)
D. Subarachnoid haemorrhage (SAH)
E. Idiopathic intracranial hypertension (IIH)

A

A. Migraine

You could convince yourself it was any of the options to be fair.

But, the lack of haemodynamic compromise, no focal neurology and no papiloedema goes against this. (D)

a Cerebral vein Thrombus is classically post-partum and will often have focal neurology (C).

The lack of an infective presentation goes against Viral meningitis (B)

(E) is often associated with young obese women but would have papilloedema.

How well did you know this?
1
Not at all
2
3
4
5
Perfectly
24
Q

A 19-year-old woman in her first pregnancy is admitted to the labour ward with a 4-hour history of lower abdominal pain – she is 22 weeks pregnant. She has not had any vaginal bleeding but describes a possible history of rupture of her membranes. Her past medical history includes an appendectomy and a large cone biopsy of her cervix. On examination she has palpable lower abdominal tenderness, her cervix is 2 cm dilated, she has an offensive vaginal discharge and her temperature is 38.9ºC. Her white cell count is 19.0 × 109/L and her C-reactive protein is 188 mg/L. There are no signs of cardiovascular compromise. How would you manage this woman?

A. Insert a cervical suture
B. 12 mg betamethasone, atosiban for tocolysis and antibiotics
C. Head down, bed rest, antibiotics and await events
D. Antibiotics and induce labour
E. Caesarean section

A

The Large cone biopsy puts her at risk of cerviacal compromise - PROM.

There is a sceptic picture here also, this combined with the dilated cervix indicates a sceptic miscarrige. option A on it’s own would not deal with the presenting scenario and is contraindicated in the case of infection.

At 22 weeks the foetus is not viable and so there is no role for steriods (B) tocolytics is contraindicated in chorioamnionitis.

The woman needs antibiotics and to have the focus of infection removed - this is option D.

Watching and waiting is not appropriate due to the severity of the case (C)

Caesarean is not advisable in a 22 week gestation.

How well did you know this?
1
Not at all
2
3
4
5
Perfectly
25
Q

A 24-year-old multiparous woman is 23 weeks pregnant. She has not had chicken pox before. She goes to a collect her 3-year-old son from a birthday party and comes into contact with a child with an infective chicken pox infection. She is naturally very anxious. What is the best course of management?

A. Wait and see if she develops a rash. If she does treat with aciclovir
B. Test for varicella antibodies and give varicella zoster immunoglobulin
(VZIG) within the first 24 hours
C. Test for varicella antibodies and give aciclovir within the first 24 hours
D. Test for varicella antibodies and give VZIG within 10 days
E. Reassure that there is no significant risk at present as contact was so brief

A

D. Test for varicella antibodies and give VZIG within 10 days

women who contract chicken pox during pregnancy are at risk of a more significat effect on themselves as well as the risk of fetal varicella syndrome if the exposure is before 28wks. This syndrome includes eye defects hypoplasia of the limbs and neurological defects.

There has been significant contact in this case, and her 3 year old may well be incubating the virus, so options A and E are inappropriate.

Although she may not think she has had chicken pox, she may have developed a sub-clinical infection previously. testing for antibodies will guide your management. Aciclovir can be started within 24hours of the RASH APPEARING so C is not correct.

Becuase of the cost involved you don’t start VZIg straight away in case she is immune (excludes B). The guidance is that VZIg is effective within 10 days.

How well did you know this?
1
Not at all
2
3
4
5
Perfectly
26
Q

A 32-year-old woman in her third pregnancy is 37 weeks pregnant and has an
extended breech baby on ultrasound. After discussion in the antenatal clinic,
which of the following is not an absolute contraindication to an external cephalic
version (ECV)?

A. Multiple pregnancy
B. Major uterine abnormality
C. Antepartum haemorrhage within 7 days
D. Rupture of membranes
E. Small for gestational age with abnormal Doppler scan

A

E. Small for gestational age with abnormal Doppler scan

I gon’t the wrong idea with answer A and thought it meant multiparous as oppossed to twin/triplet etc. Obviously that’s contra indicated.

Bleeding (C), major abnormalities (B), and a lack of fluid to protect the baby (D), are all absolute contra indications

(E) is only a relative contra indication

How well did you know this?
1
Not at all
2
3
4
5
Perfectly
27
Q

A 24-year-old type 1 diabetic woman has just had her first baby delivered by
caesarean section at 35 weeks due to fetal macrosomia and poor blood sugar
control. The operation is straightforward with no complications. She has an insulin
sliding scale running when you review her on the ward 12 hours postoperatively.
She has begun to eat and drink. How would you manage her insulin requirements?

A. Continue the sliding scale for 24 hours
B. Change her back to her pre-pregnancy insulin and stop the sliding scale
C. Halve the dose of insulin with each meal for the next 48 hours
D. Stop the insulin now that baby is delivered
E. Sliding scale for 48 hours to prevent hyperglycaemia

A

B. Change her back to her pre-pregnancy insulin and stop the sliding scale

Stopping the insulin (D) is a bit mad as she’s a Type 1…

As soon as the patient is eating there is no need to continue the sliding scale (A or E)

Halving the dose is likely to not give her enough control (C)

The only sensible option is to return to the normal insulin dose but continue to monitor.

How well did you know this?
1
Not at all
2
3
4
5
Perfectly
28
Q

A 19-year-old woman is referred to your pre-conception clinic. She has SLE
and wants to fall pregnant. She is currently not on any treatment and has no
symptoms. As part of your general counselling you should talk about the risks
associated with pregnancy. Which of the following is not a particular risk to a
woman with SLE?

A. Fetal growth restriction
B. Diabetes mellitus
C. Pre-eclampsia
D. Stillbirth
E. Preterm delivery

A

B. Diabetes mellitus

Pregnancy increases the chance of a flare up of SLE by 40-60%.

it carries the risk of , spontaneous miscarriage, Fetal Death (D), Pre-eclampsia (C), pre-term delivery (E) and fetal growh restriction (A). All of which make sense as SLE is a systemic disease of the connective tissue. THere is no reason why a connective tissue disease would affect an endocrine system as in Diabetes Mellitus (B)

How well did you know this?
1
Not at all
2
3
4
5
Perfectly
29
Q

A 44-year-old women who is 18 weeks pregnant presents to your clinic with
a 2-day history of a viral illness. She is extremely anxious and is in floods of
tears. She recently had some soft cheese in a restaurant and after an internet
search she is convinced she has a particular infection. What infection is she
concerned about?

A. Toxoplasmosis
B. Cytomegalovirus (CMV)
C. Listeria monocytogenes
D. Hepatitis E
E. Parvovirus B 19

A

C. Listeria monocytogenes

A - Toxoplasmosis is associated with cats, cat faeces, or unwashed fruit and veg

B - Is commonly sub-clinical and found in many people. It’s only an issue in the immunocompromised. No food association

C - this is associated with soft cheese, pate or unpasterised milk. Causes mid-trimester loss, early meconium and pre-term labour. Flu like illness

D - Non-chronic hepatitis, transmitted by faeco-oral route

E - Parvovirus B19 casuses the facial rash of slapped cheek syndrome/fith disease called erythema infectiosum. It has a respiratory droplet route of transmission.

How well did you know this?
1
Not at all
2
3
4
5
Perfectly
30
Q

A 26-year-old woman is 37 weeks pregnant and consults you about a rash that
started on her abdomen and has now spread all over her body. Interestingly her
umbilicus is spared. The rash is very itchy and nothing is helping. The rash is
her first problem in this pregnancy. Of interest, her mother has pemphigoid and her sister has psoriasis. What is the most likely cause of her rash?

A. Pemphigoid gestationis
B. Pruritic urticarial papules and plaques of pregnancy (PUPP)
C. Impetigo herpetiformis
D. Prurigo gestationis
E. Contact dermatitis

A

B. Pruritic urticarial papules and plaques of pregnancy (PUPP)

Pemphigoid gestationis is a blistering condition that starts at teh umbilicus and then spreads.

Impetigo herpetiformis is a febrile blistering condition that can lead to maternal and fetal death.

Prurigo gestationis is a rash of the uper arms and trunkwith abdominal sparing

Contact dermatitis is possible if there was a history of use of lotions or creams but unless there was a reason in the history it would not be sparing the umbilicus.

How well did you know this?
1
Not at all
2
3
4
5
Perfectly
31
Q

Which of the following drugs is not absolutely contraindicated in pregnancy?

A. Acitretin
B. Fluconazole
C. Mebendazole
D. Sodium valproate
E. Methotrexate

A

D. Sodium valproate

Valporate should be avoided as it has the highest risk of congenital malformations of all the anti convulsants. But if it is the most appropriate agent for the patient it should be used.

Acitretin is a retanoid and methotrexate is a chemical pregnancy termination agent (amougst other uses) these are both obviously a big no-no.

Mebendazole is an anti worm tablet and has been shown to have toxic effects in animal studies

Fluconazole is an anti fungal and is linked with many congenital abnormalities.

How well did you know this?
1
Not at all
2
3
4
5
Perfectly
32
Q

A 42-year-old woman is in her first pregnancy. She conceived with in vitro
fertilization (IVF) and has had a straightforward pregnancy so far. At 25 weeks’
gestation she is seen in clinic with a blood pressure of 142/94 mmHg and protein +
in her urine. A protein creatinine ratio (PCR) comes back as 19. She says that her
blood pressure is often up at the doctor’s. With the information you have to hand
what is the most likely diagnosis?

A. Pre-eclampsia
B. White coat hypertension
C. Essential hypertension
D. Conn’s syndrome
E. Pregnancy-induced hypertension

A

E. Pregnancy-induced hypertension

(B) is a diagnosis of exclusion and shouldn’t be resorted to without another identifiable cause

(A) Pre-eclampsia is a diagnosis of raised blood pressure and proteinuria, a PCR of less than 30 is considered normal. So this is not the diagnosis in this case

(D) there’s nothing here to suggest Conn’s

The fact that the hypertension has been identified in the context of pregnancy makes this (E) not (C).

How well did you know this?
1
Not at all
2
3
4
5
Perfectly
33
Q

A 24-year-old woman attends the antenatal clinic. She has had a glucose tolerance test which is abnormal. A diagnosis of gestational diabetes is made. The primary
purpose of this appointment is to explain to her what gestational diabetes means
to her and her baby. You explain to her that sugar control is important and there
are specific glucose ranges that she should try to adhere to. Which of the following
would be correct advice for this woman?

A. Pre meal blood sugar <7.1 μmol/L
B. Post meal 1-hour sugar <11.1 μmol/L
C. Post meal 1-hour sugar <7.8 μmol/L
D. Post meal 2-hour blood sugar <7.8 μmol/L
E. Pre meal blood sugar <7.8 μmol/L

A

C. Post meal 1-hour sugar <7.8 μmol/L

Gestational diabetes mellitus is diagnosed as a blood sugar of greater than 7.8 μmol/L one hour after the oral glucose test. As such this is the control level to be aimed for. The pre-meal blood sugars should be less than 5.5μmol/L

How well did you know this?
1
Not at all
2
3
4
5
Perfectly
34
Q

A 24-year-old woman in her first pregnancy has a significantly raised glucose
tolerance test at 28 weeks gestation: 4.6 fasting 12.1 at one hour 9.1 at 2 hours
(μmol/L). She is given the diagnosis of GDM. You are asked to counsel her about
the effects of gestational diabetes on pregnancy. Which of the following is not an
additional effect of having GDM?

A. Shoulder dystocia with a macrosomic fetus
B. Stillbirth
C. Neonatal hypoglycaemia
D. 10 per cent chance of developing type 2 diabetes over the next
10 years
E. Pre-eclampsia

A

D. 10 per cent chance of developing type 2 diabetes over the next
10 years

the risk of developing DM type 2 is actually 35-60% over the next 10-15 years.

All the other statements are correct

How well did you know this?
1
Not at all
2
3
4
5
Perfectly
35
Q

A 24-year-old woman who is HIV positive is in her first pregnancy. She is 39
weeks pregnant and is seen by you in the antenatal clinic. She has just transferred
to your care, with no other previous antenatal care. She reports that her pregnancy
has been uncomplicated. Her CD4 count is 180/mm3 and her viral load is 5500
copies/mL. She has come to find out what advice you have for her delivery.

A. Spontaneous vaginal delivery
B. Induction of labour to prevent CD4 decreasing
C. Caesarean section
D. Start highly active antiretroviral therapy (HAART) and await for labour
to start
E. Start HAART, amniotomy and HAART for baby when born

A

C. Caesarean section

Due to this woman’s viral load being high we want to avoid vertical transmission, caesarean section has evidence that it does reduce the risk.

For the same reasons prolonged rupture of membranes and any artificial rupture should be avoided (B and E)

A normal delivery can be advised, but only if the viral load is undetectable (less than 50 copies/ml) this excludes options A and D in this case.

N.B. Amniotomy = artificial rupture of membranes

How well did you know this?
1
Not at all
2
3
4
5
Perfectly
36
Q

A 24-year-old woman attends accident and emergency 4 weeks after having a
positive urinary pregnancy test. She has had 3 days of painless vaginal bleeding
and is passing clots. Over the past 2 days the bleeding has settled. An ultrasound
scan shows an empty uterus. What is the correct diagnosis?

A. Threatened abortion
B. Missed miscarriage
C. Septic abortion
D. Complete abortion
E. Incomplete miscarriage

A

D. Complete abortion

Threatened abortion = any bleeding before viability (24 weeks currently). after this point bleeding is ante-partum haemorrhage

Missed Miscarriage = The loss of a pregnancy without the passage of products of conception or bleeding.

Septic Abortion = The loss of a pregnancy with evidence of infection, the infection is of the retained conceptus. must be managed actively as there is still a high mortality

Incomplete abortion = The loss of a pregnancy with bleeding and/or passage of some, but not all, products of conception. it can be managed conservatively, medically (misoprostol), or through ‘surgical management of miscarriage’ (previously called evacuation of retained products of conception, ERPC)

How well did you know this?
1
Not at all
2
3
4
5
Perfectly
37
Q

A 51-year-old woman in her 12th week of an assisted-conception triplet pregnancy
presents to accident and emergency with severe nausea and vomiting. She has mild
lower abdominal and back pains. Urine dipstick shows blood –ve, protein –ve,
ketones ++++, glucose +. What is the most appropriate management plan?

A. Intravenous crystalloids and doxycycline, urgent ultrasound assessment
B. Discharge with 1 week’s course of ciprofloxacin
C. Referral to the medics for investigation of viral gastroneteritis
D. Intravenous crystalloids, oral antiemetics
E. Referral to the surgeons for investigation of appendicitis

A

D. Intravenous crystalloids, oral antiemetics

The triplet pregnancy is a risk factor for the development of Hyperemesis gravidarum. The ketonuria also points to this diagnosis as a indicator of dehydration.

The mild back and abdo pains are normal for this stage in pregnancy.

The management of HG is to rehydrate and provide antiemetics (D)

Without diarrhoea or sepsis we shouldn’t be leaning towards viral gastroenteritis or appendicitis (C, E), and it’s certainly not appendicitis with a mild pain.

Tetracyclines (Doxy) and Quinolones (cipro) are both teratogenic

How well did you know this?
1
Not at all
2
3
4
5
Perfectly
38
Q

A 19-year-old woman is referred to your early pregnancy unit as she is having
some vaginal bleeding. This is her first pregnancy, she has regular menses and the
date of her last menstrual period suggests she is 8 weeks gestation today. She is
well apart from her bleeding and is naturally concerned. A transvaginal ultrasound
reveals an intrauterine gestational sac of 18 mm with a yolk sac. What is the most
likely explanation of these findings?

A. A viable intrauterine pregnancy
B. A pseudosac
C. A blighted ovum
D. A pregnancy of uncertain viability
E. An anembryonic pregnancy

A

D. A pregnancy of uncertain viability

Vaginal bleeding in early pregnancy is common but not normal, it has a miscarrige rate of 20%. This scenario by definition is a threatened miscarrige.

Without a fetal pole or a pulsatation we canot say for certain that this is a viable pregnancy (A)

At 6 weeks we would expect to see a heartbeat, but the dates may be wrong.

C and E are the same thing, it is a gestational sac without a fetal pole or yolk sac. There is a yolk sac in this case.

A psudosac (B) is the development of decidual reaction in an ectopic pregnancy, there would be no yolk sac, and this isn’t ectopic.

How well did you know this?
1
Not at all
2
3
4
5
Perfectly
39
Q

A 31-year-old woman is seen in the termination of pregnancy (TOP) clinic requesting a termination. She is 5 weeks pregnant in her first pregnancy. She is otherwise well but does have some lower abdominal pain on the right hand side. On examination her abdomen is soft and non-tender. An ultrasound reveals a small sac in the uterus which might be a pseudosac. What would be your next management step?

A. Urgent referral to hospital to rule out ectopic pregnancy
B. Rescan in 10 days time
C. Blood test for beta human chorionic gonadotrophin (hCG) now and in
48 hours time
D. Arrange for her to come in for a medical termination
E. Arrange a surgical termination of pregnancy

A

C. Blood test for beta human chorionic gonadotrophin (hCG) now and in
48 hours time

If the sac in the uterus is a psudosac then it points to an ectopic pregnancy, this is supported by the RIF pain.

The woman is, however, heamodynamically stable and has a SNT abdomen so an urgent referral is not needed at this stage (A) as long as she is able to understand red flag symptoms.

She needs a bhCG level now and then again in 48hrs. If there is a 67% rise in bhCG then it indicates that the sac in the uterus is a viable pregnancy, and following an ultrasound scan to confirm in ten days, a TOP can be offered to this woman if she wishes.

If the rise in bhCG is less than this she should be seen for further assesment in hospital

An interuterine pregnancy should always be confirmed prior to offering a TOP.

How well did you know this?
1
Not at all
2
3
4
5
Perfectly
40
Q

A 28-year-old woman with a history of pelvic inflammatory disease is 6 weeks
into her third pregnancy. She previously had two terminations. She presents with
lower abdominal pain and per vaginam bleeding. Her beta hCG is 1650 mIU/mL,
progesterone 11 nmol/l. An ultrasound reveals a small mass in her left fallopian
tube with no intrauterine pregnancy seen. There is no free fluid in the Pouch of
Douglas. She is diagnosed with an ectopic pregnancy and is clinically stable but
scared of surgery. How would you manage this case?

A. Laparoscopic salpingectomy
B. Methotrexate
C. Laparotomy + salpingectomy
D. Laparoscopic salpingotomy
E. Beta hCG in 48 hours

A

B. Methotrexate

This woman has risk factors for an ectopic pregnancy, PID, and previous terminations. Other risks include tubal surgery, previous ectopics and IVF.

A salpingectomy would be definitive but is an extreme step to leave a 28 year old woman with one tube. An open [rocedure is definitely not needed as she is stable.

A salpingotomy (opening the tube but leaving in in situ) may just increase the risk of further ectopic pregnancys.

The criteria for methotrexate use in an ectopic are that it is a small ectopic with no fetal pulse, no clinical compromise, and no free fluid on the pouch of Douglas. This woman is an ideal candidate.

Monitoring the bhCG in 48hrs would indicate if this pregnancy is failing spontaneously. Given that she is a candidate for methotrexate it is the most definitive management.

How well did you know this?
1
Not at all
2
3
4
5
Perfectly
41
Q

A 24-year-old woman attends her GP complaining of deep dyspareunia and
post-coital bleeding. She has crampy lower abdominal pain. Of note, she has
been treated in the past for gonorrhoea on more than one occasion. On speculum
examination there is no visible discharge, but the cervix bleeds easily on contact.
What is the most appropriate management?

A. IM cefotaxime, oral doxycycline and metronidazole
B. 1 g oral metronidazole stat
C. Urgent referral to the gynaecology clinic
D. Referral to a sexual health clinic
E. Admission to hospital under the gynaecologists

A

C. Urgent referral to the gynaecology clinic

The easily bleeding cervix is the alarming factor here. The woman’s significant history of sexually transmitted infections could lead you to think of PID or another infection, for which A,B,D and E are all management options, the cervix bleeding easily is a concern for cervical cancer. At the age of 24 she is not included in the cervical screening program

This concern should be investigated urgently (C)

How well did you know this?
1
Not at all
2
3
4
5
Perfectly
42
Q

16-year-old girl attends accident and emergency complaining of mild vaginal
spotting. Her serum beta hCG is 4016 mIU/mL. She is complaining of severe leftiliac fossa pain and stabbing sensations in her shoulder tip. What is the most
appropriate definitive investigation?

A. Diagnostic laparoscopy
B. Serial serum beta hCG measurement
C. Computed tomography of the abdomen and pelvis
D. Clinical assessment with speculum and digital vaginal examination
E. Transvaginal ultrasonography

A

E. Transvaginal ultrasonography

The severe LIF pain and shoulder tip pain strongly indicates an ectopic pregnancy that has potentially ruptured and the free fluid is irritating the diaphram and causing the reffered shoulder tip pain.

An early pregnancy should be visable if the bhCG is above a 1000 so a TVUS should see an interuterine pregnancy if this sisn’t an ectopic.

The Laparoscopy (A) would only be first line if she was unstable. At this stage it should be performed if the TVUS confirms an ectopic.

CT is too teratogenic (C)

The serial bhCG (B) would only give useful information after 48hrs, given the picture it’s not wise to wait that long.

Clinical assesment (D) would not be definitive

How well did you know this?
1
Not at all
2
3
4
5
Perfectly
43
Q

An 18-year-old woman presents to accident and emergency having fainted at work. She is complaining of pain in the lower abdomen. A serum beta hCG performed in the emergency department is 3020 mIU/mL. The on-call gynaecologist performs transvaginal ultrasonography in the resuscitation area which shows free fluid in the Pouch of Douglas and no visible intrauterine pregnancy. Her pulse is 120 bpm and blood pressure 90/45 mmHg. What is the most likely diagnosis?

A. Ruptured ovarian cyst
B. Cervical ectopic pregnancy
C. Ruptured tubal pregnancy
D. Perforated appendix
E. Ovarian torsion

A

C. Ruptured tubal pregnancy

  • collapse and circulatory compromise (low BP, tachycardia)
  • A bhCG over 1000 would be a visible pregnancy in the uterus on TVUS, it is not visable. Therefore ectopic
  • Fluid in the pouch of douglas indicates a degree of rupture

The ruptured cyst (A) and torsion (E) would both have sudden onset but no haemodynamic compromise. of the two the cyst would have pain that improves with simple analgesia and time, the torsion would not get better.

How well did you know this?
1
Not at all
2
3
4
5
Perfectly
44
Q

A 50-year-old woman comes to your clinic with a 2-year history of no periods. Her
GP has confirmed that her luteinizing hormone and follicle-stimulating hormone levels are menopausal. Her night sweats and hot flushes are unbearable and are preventing her from going to work. She would like to start hormone replacement therapy (HRT) but is very worried about the side effects. Which of the following is incorrect?

A. There is evidence that HRT prevents coronary heart disease
B. There is a small increase in the risk of strokes
C. There is an increased risk of breast cancer
D. There is an increase in the risk of ovarian cancer
E. There is an increase in the rate of venous thromboembolism

A

A. There is evidence that HRT prevents coronary heart disease

oestrogen increases the risk of ovarian and breast Ca (D and C)

There is a small increased risk of stroke (B) and a small increase in DVT risk (E)

How well did you know this?
1
Not at all
2
3
4
5
Perfectly
45
Q

A 24-year-old woman who is 9 weeks pregnant is brought to accident and
emergency by ambulance with left iliac fossa pain and a small vaginal bleed.
An abdominal ultrasound scan performed at the bedside demonstrates a cornual
pregnancy and free fluid in the pelvis. Her observations are: pulse 119 bpm, blood
pressure 74/40 mmHg, respiratory rate 24/minute. What is the most appropriate
definitive management?

A. Transvaginal ultrasound scan
B. Serum beta hCG estimation

C. Diagnostic laparoscopy
D. Admission to the gynaecology ward and fluid resuscitation
E. Urine pregnancy test

A

C. Diagnostic laparoscopy

A cornual pregnancy is a specific ectopic (see image). This patient has a confirmed ectopic and she is haemodynamically compromised, urgent curative action is needed.

A TVUS doesn’t add anything when we have a diagnosis (A)

Likewise a bhCG doesn’t change anything either, it’s purely diagnostic and we have a diagnosis already. (B)

option D is too conservative, fluid resucitation won’t improve the situation.

Finally E is the same situation, no curative effect.

How well did you know this?
1
Not at all
2
3
4
5
Perfectly
46
Q

A 26-year-old woman presents to accident and emergency with left-sided lower
abdominal pain and a single episode of vaginal spotting the day before. A
urinary beta hCG is positive, and her last period was 6 weeks ago. A transvaginal
ultrasound shows two gestational sacs. What is the most likely diagnosis?

A. Ruptured theca lutein cyst
B. Appendicitis
C. Diverticulitis
D. Complete miscarriage
E. Urinary tract infection

A

A. Ruptured theca lutein cyst

This pain has localised to the wrong side for appendicitis (B)

Diverticulitis is extremely unlikely in this age group (C)

A complete miscarriage would not have a positive bhCG or have gestational sacs on the scan (D)

A UTI would have urinary symptoms and classically would localise to the suprapubic area (E)

A functional ovarian cyst is common in women of child bearing age and a theca lutein cyst of cyst is more common in multiple pregnancy. In this case it seems to have ruptured and bleed slowly into the peritonium.

How well did you know this?
1
Not at all
2
3
4
5
Perfectly
47
Q

A 32-year-old woman with paranoid schizophrenia is admitted for antenatal
assessment at 36 weeks’ gestation with twins. Her pregnancy is complicated
by intrauterine growth restriction and impaired placental flow. She has had no
psychotic symptoms in this pregnancy. Her obstetricians recommend an early
caesarean section and argue it is in the best interests of the mother and her
babies and to prevent further fetal insult. She has repeatedly said that despite the
significant risks, which she understands, she refuses caesarean delivery. What is
the most appropriate action?

A. Detain under Section 5 of the Mental Health Act and deliver by
caesarean section
B. Detain under Section 2 of the Mental Health Act and deliver by
caesarean section
C. Determine that the patient lacks mental capacity and, acting in her best
interests, delivery by caesarean section
D. Determine that the patient lacks mental capacity and, acting in her
fetus’ best interests, deliver by caesarean section
E. Encourage volunatary admission to the antenatal and repeatedly explain
the benefits of caesarean delivery

A

E. Encourage volunatary admission to the antenatal and repeatedly explain
the benefits of caesarean delivery

The patient has made a solid case that she has capacity and that she is not suffering from an active psyciatric condition at this time. If she has capacity and is not sufferin from a mental health condition then there is no place for the MH act or the MC act.

E is the only possible option here

On a side note, in the UK you can never legally act on the best interests of a fetus.(option D)

How well did you know this?
1
Not at all
2
3
4
5
Perfectly
48
Q

Which of the following would be incorrect advice to give a woman requesting a
caesarean section for non-medical indications?

A. You are twice as likely to have a stillbirth in a subsequent pregnancy
B. The risk of damaging the bladder is one in 20
C. There is an increased risk of placenta praevia in future pregnancies
D. 1–2 per cent babies suffer lacerations
E. The risk of infection is 6 per cent

A

B. The risk of damaging the bladder is one in 20

A 5% risk is grossly exagerrated here, the real risk is 1:1000

the rest are all correct. 6% is a standar infection risk, there is a chance the baby will suffer a laceration and the scar on the uterus can lead to placenta praevia.

The incrreased risk of still birth was found in a 2014 Danish study, I couldn’t find a mechanism for this.

How well did you know this?
1
Not at all
2
3
4
5
Perfectly
49
Q

A 24-year-old Jehovah’s Witness is brought to accident and emergency with a
Glasgow coma scale (GCS) score of 3, BP 90/30 mmHg and pulse 110 bpm. Her
husband reports that her last menstrual period was 8 weeks ago and she complained this morning of lower abdominal pain and vaginal spotting. Ultrasonography suggests a ruptured ectopic pregnancy. As part of the resuscitative measures employed before emergency laparotomy, a transfusion of group O-negative blood is prepared. Her husband interrupts and says that as a Jehovah’s Witness she would absolutely refuse all blood products even at risk of death, and has previously signed an advance directive stating this. What is the most appropriate option?

A. Avoid transfusion and volume-replace with colloids before emergency
transfer to theatre
B. Avoid transfusion and use a Cell Saver auto-transfuser in theatre
C. Avoid transfusion and immediately transfer to theatre
D. Stabilize the woman in accident and emergency before transfer to
theatre
E. Transfuse the woman with group-O negative blood and immediately
transfer to theatre

A

E. Transfuse the woman with group-O negative blood and immediately
transfer to theatre

in order to be accepted a legally valid advance directive needs to be seen by the clinician in person. The husbands asertation cannot be taken in to account. Te clinicain must act in the patient’s best interests.

The woman is severly compromised and needs resucitation with blood, any avoidance of transfusion completely (B and C) is completely out.

Blood is the best option to stabilise prior to immediate transfer to theatre.

How well did you know this?
1
Not at all
2
3
4
5
Perfectly
50
Q

An unbooked 26-week pregnant woman sees you at the hospital to request
a termination of pregnancy. She says that if she leaves here today without
a termination she will try and do it herself by stabbing her abdomen. Your
consultant arranges an urgent psychiatric review which finds no grounds
under which to detain this woman in regards to her mental health. Under these
circumstances, if a termination was performed, which part of the Abortion Act
would it fall under?

A. The continuance of the pregnancy would involve risk to the life of the
pregnant woman greater than if the pregnancy were terminated
B. The termination is necessary to prevent grave permanent injury to the
physical or mental health of the pregnant woman
C. The pregnancy has not exceeded its 24th week and continuance of
the pregnancy would involve risk, greater than if the pregnancy were
terminated, of injury to the physical or mental health of the pregnant
woman;
D. The pregnancy has not exceeded the continuance of the pregnancy and
would involve risk, greater than if the pregnancy were terminated, of
injury to the physical or mental health of any existing child(ren) of the
family of the pregnant woman
E. There is a substantial risk that if the child were born it would
suffer from such physical or mental abnormalities as to be seriously
handicapped.

A

B. The termination is necessary to prevent grave permanent injury to the
physical or mental health of the pregnant woman

answers C and D refer to terminations before 24 weeks and so are out completely.

There is no evidence here to support option E.

We are at this point left with A or B as options. Legally speaking it is easier to justify that the woman will cause grave harm to herself in this scenario, there is probably a risk of death but it is not as likely as grave harm. For this reason B is the correct answer, although this is a seriously dodgy scenario, terminating a potentially viable fetus.

How well did you know this?
1
Not at all
2
3
4
5
Perfectly
51
Q

A 24-year-old woman in her first pregnancy presents to the labour ward in
labour. She and her partner express an overwhelming desire to avoid a caesarean
section. Her labour does not progress and after 9 hours her cervix is still only
3 cm dilated. Unfortunately, the fetal heart slows to 60 beats and does not recover
after 5 minutes. Your senior registrar explains the situation to the woman and
recommends an immediate caesarean section. She refuses and her partner tells
you to stop harassing them. You explain that their unborn child will die if this
continues. What options do you have?

A. Caesarean section under general anaesthesia (GA) under Section 3 of the
Mental Health Act
B. Caesarean section under GA under Section 2 of Mental Health Act
C. No action. Allow fetus to die
D. Caesarean section without Mental Health Act application
E. Caesarean section under GA under Section 5(2) of the Mental Health Act

A

C. No action. Allow fetus to die

This one is grimly straigtforward. The woman does not appear to be suffering from a mental health condition and so the MH act would not be appropriate. THis act also only convers treatment for mental health conditions.

Any caesarean without consent would be a criminal act, unfortunately answer C is the only option here.

How well did you know this?
1
Not at all
2
3
4
5
Perfectly
52
Q

A 16-year-old Muslim woman attends accident and emergency department with
her father. She complains of a 1-day history of left iliac fossa pain and mild
vaginal spotting. A urinary beta hCG test is positive. As part of your assessment
the patient consents to a vaginal examination. She insists you do not tell her father
that she is pregnant, and you consider her to be competent in her judgement. Her
father becomes angry and says you must not perform a vaginal examination. How
should you proceed?

A. Perform the examination with a chaperone present and tell the father
that it is a routine examination
B. Perform the examination with a chaperone present and explain that
parental consent is not necessary in this situation
C. Defer performing the examination and document the situation fully
D. Perform the examination with a chaperone present having assessed the
girl’s Gillick Competence
E. Perform the examination with a chaperone present having assessed the
girl’s Fraser Competence

A

B. Perform the examination with a chaperone present and explain that
parental consent is not necessary in this situation

The girl is over 16 so is considered competent to consent to a procedure as per the same criteria as an adult. This means that any Gillick/Fraser considerations are not relevant. (D and E)

A necessary clinical examination should not be delayed due to the father’s (actually irrelevant) viewpoint.

At this point I disagree with the book answer of B, normalising the examination and performing the exam with a chaperone seems the less confrontational option here (A). The book argues that option A breeches the girl’s confidentiality but I do’t see how that is true.

How well did you know this?
1
Not at all
2
3
4
5
Perfectly
53
Q

A 32-year-old woman is rushed to accident and emergency as the viction of a
high speed vehicle collision. She is 35 weeks pregnant and unconscious. There
is evidence of blunt abdominal trauma and she is showing signs of grade 3
hypovolaemic shock. The consultant obstetrician on call immediately attends
the resus call and recommends immediate perimortem caesarean delivery in a
resuscitative effort to improve the management of her shock. Her husband has
been brought into resus by the police, and insists that she would refuse caesarean
section under any circumstances. What is the most appropriate management?

A. Rapid fluid resuscitation until the situation regarding the patient’s
wishes becomes clear
B. Replacement of the lost circulating volume with blood products
C. Admit to the intensive care unit and begin infusing inotropes to restore
the cardiac output
D. Immediate caesarean delivery
E. Resucitation and transfer to the obstetric theatre for emergency
caesarean delivery

A

D. Immediate caesarean delivery

In such an emergency the best interests of the patient is all that matters, the husband’s wishes are irrelevant. The patient’s wishes are unlikely to become any clearer in the time she has remaining (A)

Replacing with fluid (A) or blood (B) will only buy a little time

Transfering the patient to theatre (E) or ITU (C) is very risky as the patient may arrest en route. The use of inotropes (C) will only partially help as it won’t replace lost volume.

The primary aim of a perimortem caesarean delivery is to save the mother’s life. The procedure reduces the volume of the gravid uterus which reduces venous return and so pre-load on the heart.

How well did you know this?
1
Not at all
2
3
4
5
Perfectly
54
Q

A 59-year-old woman has been admitted for a hysterectomy for endometrial cancer.
She has not yet given her consent and the rest of the team is in theatre. You have
performed a hysterectomy before so feel confident in taking her through what will
happen and the risks involved. The General Medical Council (GMC) says that you
should tailor your discussion to all of the options except which of the following?

A. Their needs, wishes and priorities
B. Their level of knowledge about, and understanding of, their condition,
prognosis and the treatment options
C. The onset of their condition
D. The complexity of the treatment
E. The nature and level of risk associated with the investigation

A

C. The onset of their condition

There are several considerations in regards to assesing how to provide information. A,B,D, and E are all legitimate considerations. Option C is an incorrect statement and should be ‘the nature of thier condition’

Without these five considerations there is an argument that a patient is not fully informed, and thus cannot give adequate consent.

How well did you know this?
1
Not at all
2
3
4
5
Perfectly
55
Q

A quality improvement process that seeks to improve patient care and outcomes
through systematic review of care against explicit criteria and the implementation
of change. This is an accepted definition of what?

A. Audit
B. Clinical governance
C. Clinical research
D. Clinical effectiveness
E. Integrated governance

A

A. Audit

This is a descrition of an audit.

Clinical governance is a wide ranging term that covers audit, researchand a framework for care care improvement

Clinical Reasearch is research that seeks to answer a specific scientific question

Intergrated governence is the combination of clinical and financial governance

Clinical effectiveness is clinical research that also assesses if the novel treatment is acceptable to patients and represents good value for money.

How well did you know this?
1
Not at all
2
3
4
5
Perfectly
56
Q

A 15-year-old girl attends the gynaecology clinic with her boyfriend, also 15,
requesting the morning after pill 4 months after being circumcised during a family
trip to Somalia. She understands your advice and the implications of her decisions
to engage in sexual activity, is using condoms regularly and refuses to inform her
parents. What is the most appropriate management?

A. Decline to prescribe the morning after pill and refer the patient back to
her GP
B. Decline to prescribe the morning after pill, and inform her parents that
she is having underage sex
C. Prescribe the morning after pill, give contraceptive advice and
recommend that the girl informs her parents
D. Prescribe the morning after pill, give contraceptive advice and
immediately alert your consultant and the Safeguarding Children Team
E. Prescribe the morning after pill, give contraceptive advice and inform
her parents

A

D. Prescribe the morning after pill, give contraceptive advice and
immediately alert your consultant and the Safeguarding Children Team

This girl fulfills the fraser guidelines in that she understands the advice given, cannot be persuaded to tell her parents and is likely to continue having sexual intercourse, which will cause her health to suffer without contraception being provided. So options A and B are immediately incorrect.

You cannot tell her parents as this is a breach of confidentiality (E)

This leaves options C and D, but only D addresses the issue of Female ‘circumcision’, more correctly termed female genital mutilation in this country. This is a serious offece under UK law and needs to be addressed.

57
Q

A 59-year-old woman attends the gynaecology clinic complaining of worsening
pain during penetrative sexual intercourse. She went through the menopause
9 years before, with very few problems, and did not require hormone replacement
therapy (HRT). She has been with the same partner for 4 years since the death of
her husband with whom she had four children. What is the most likely diagnosis?

A. Ovarian malignancy
B. Chlamydia trachomatis infection
C. Discoid lupus erythematosus
D. Atrophic vaginitis
E. Bacterial vaginosis

A

D. Atrophic vaginitis

Dyspareunia is pain on intercourse, either deep or superficial.With any dyspareunia it is important to rule out a sexually trasmitted infection, intrapelvic conditions (appendicitis, ovarian pathology), or cervical pathology.

The vaginal lining of post menopausal women gradually atrophies without the influence of eostrogen.

STD is uncommon (but not unplausabile) in this age group

pH changes in the vagina rule out BV in this age group (E)

malignancy (A) is not sugessted by this presentation.

Discoid Lupus (C) is unlikely as there are no features here of such a systemic disease, the age of onset would also be younger (in her 30’s)

58
Q

A 19-year-old woman is referred to accident and emergency with a fluctuant lower
right abdominal pain which started over the course of the morning, associated with
vomiting. There is rebound tenderness on examination. She is afebrile. Serum beta
human chorionic genadotrophin (hCG) is negative. An ultrasound shows free fluid in the peritoneal cavity but no other pathology to account for the pain. White cells are 14 × 109/L and the C-reative protein (CRP) is 184 mg/L. What is the most likely diagnosis?

A. Acute appendicitis
B. Early ectopic pregnancy
C. Pelvic inflammatory disease (PID)
D. Tubo-ovarian abscess
E. Ovarian torsion

A

A. Acute appendicitis

It’s important ot be able to distinguish between gynecological and surgical issues as in this case.

The negative beta hcG rules out an ectopic (B)

Ovarian torsion (E) would present with sudden pain that was completely unrelenting and classically only responds to opioid analgesia. The pain in this case is fluctuant, and there is a leukocytosis.

Tubo-ovarian abscess (D) and PID (C)would present with a fever as well as the leukocytosis. The abcess patient would also be much more significantly unwell than this patient.

59
Q

A 39-year-old woman is seen in the gynaecology clinic having been diagnosed
with polycystic ovarian syndrome (PCOS). She has lots of questions in particular
about the associated long-term risks. Which of the following is not a risk of PCOS?

A. Endometrial hyperplasia
B. Sleep apnoea
C. Diabetes
D. Breast cancer
E. Acne

A

D. Breast cancer

PCOS is a diagnosis based on 2 of 3 of; oligo/amennhorea, polycystic ovaries, and clinical and/or biochemical signs of hyperandrogenism.

Everything listed except (D) is a risk of PCOS.

There is no link between PCOS and breast or ovarian cancer.

60
Q
A 54-year-old menopausal woman comes to your clinic desperate for hormone
replacement therapy (HRT) as her vasomotor symptoms are very troubling. Her
next door neighbour recently developed a deep vein thrombosis while on HRT. She is concerned about the risks of venous thromboembolism (VTE) and wants your advice. Which of the following would you not advise?

A. The risk of VTE is highest in the first year of taking HRT
B. She should have a thrombophilia screen prior to starting HRT
C. There is no evidence of a continuing VTE risk after stopping HRT
D. Personal history of VTE is a contraindication to oral HRT
E. If she develops any VTE while on HRT it should be stopped immediately

A

B. She should have a thrombophilia screen prior to starting HRT

A thombophillia screen is not offered routinely to women starting HRT as it’s not cost effective. With a family history of VTE it would probably be sensible.

All the other points here are correct.

61
Q

A 34-year-old woman with long-standing menorrhagia attends accident and
emergency having fainted at home. She is on the third day of her period, which has
been unusually heavy this month. She insists she cannot be pregnant as she has
not had sexual intercourse for a year. She is haemodynamically stable. A point-ofcare
test venous full blood count in the emergency department shows:
Hb 5.2 g/dL
WCC 8.9 × 109/L
Hct 0.41% L
MCV 80 fL
What should the initial management be?

A. Establish large-bore venous access, commence fluid resuscitation and
cross-match four units of packed red cells
B. Call for senior help, establish large-bore venous access and prepare the
patient for urgent laparotomy
C. Call for senior help, establish large-bore venous access and give group
O rhesus negative blood
D. Establish large-bore venous access and begin transfusing group-specific
blood as soon as it is available
E. Await the result of a beta hCG test before deciding further management

A

A. Establish large-bore venous access, commence fluid resuscitation and
cross-match four units of packed red cells

This woman has a normocytic anaemia as a result of blood loss. Her history rules out pregnancy (ectopic) and so the approach in (B) is inappropriate.

She should be apropriately resuscitated straight away so the delay invloved in (E) and (D) is inapropriate.

She may decompensate rapidly so a large bore cannula is needed.

At this stage teh risk of giving a blood product is not justified and so fluids is the safest initial management while units are cross matched (takes about 45 mins)

62
Q

A 66-year-old post-menopausal woman is referred to you urgently by her
general practioner (GP). She had been complaining of some lower abdominal
pain. An ultrasound arranged by the GP shows a 4 cm simple left ovarian
cyst. A CA 125 comes back as 29 U/ml (normal 0-35 U/ml). What is the most
appropriate management?

A. Referral to a specialist cancer unit
B. Laparoscopic ovarian cystectomy
C. Laparotomy and oophrectomy
D. Conservative management
E. Total laparoscopic hysterectomy and bilateral salpingo-oophorectomy

A

D. Conservative management

There are several factors to consider with risk stratifying an ovarian cyst. Features that are concerning for malignancy include; bilateral cysts, multiloculated, solid components, ascites and metastases. This is combined with teh Ca125 to give a risk of malignancy index (RMI).

In this case the cyst is simple and less than 5cm, there is also a ca125 less than 30U/ml. This means that conservative managemtn is the best course of action, this will involve 4-monthly scans and ca125 levels for a year.

63
Q

A 79-year-old woman attends your clinic with some vaginal bleeding. Her last
period was 16 years ago. She has had two children both via caesarean section,
has a normal smear history and is currently sexually active. On examination the
vagina appears mildly atrophic with some raw areas near the cervix. What is the
most important next step in her management?

A. Vagifem nightly for 2 weeks and then twice a week after that
B. Triple vaginal swabs for sexually transmitted infection
C. Pelvic ultrasonography
D. HRT to help the vaginal raw areas
E. Smear test

A

C. Pelvic ultrasonography

Post menopausal bleeding must always be investigated for cancer. In this patient’s case the smear history is normal and she is past the age range for screening so it is unlikely to be cervical (E). Endometrial cancer must be ruled out with an ultrasound. (C)

She is sexually active and so triple swabs (B) is a sensible part of the plan but not the first step.

This woman has no other menopausal symptoms and so HRT is not indicated. (D)

Vagifem (A) might be appropriate but endometrial cancer must be ruled out before prescribing oestrogen.

64
Q

At laparoscopy a 21-year-old woman is found to have severe endometriosis. There
are multiple adhesions and both ovaries are adherent to the pelvic side wall. The
sigmoid colon is adherent to a large rectovaginal nodule. The nodule is excised
and the bowel and ovaries freed. Which of the following medications would be
appropriate to help treat her endometriosis?

A. Danazol
B. Triptorelin
C. Microgynon 30
D. Tranexamic acid
E. Medroxyprogesterone acetate

A

B. Triptorelin

Danazol (A) is an anti-oestrogenic and anti-progesteronic, it is licenced for 3-6 months

Triptorelin (B) is a GnRH agonist that induces a temporary artificial menopause. it is an excellent option for up to 6 months before bone densisty loss becomes an issue.

Microgynon 30 (C) is a combined oral contraceptive

Tranexamic acid (D) is an antifibrinolytic used for menorrhagia.

Medroxyprogesterone acetate (E) is a progestogen.

This question relies on you knowing the management for this individual patient. She needs to be on medical management and then have a second laparoscopy to remove any residual disease. Triptorelin is the best option for this.

65
Q

A 54-year-old woman comes to your clinic complaing of hot flushes and night sweats that are unbearable. Her last mentrual period was 14 months ago. She has had a levonorgestrel releasing intrauterine system (Mirena) in situ for 2 years as treatment for extremely heavy periods. What treatment would you consider for her symptoms?

A. Elleste Solo
B. Elleste Duet
C. Vagifem
D. Oestrogen implants
E. Evorel

A

A. Elleste Solo

The abscence of periods for over 12 months indicates this woman is menopausal. Her symptomatic issues here require the consideration of HRT, which requires a careful examination of her history.

If the woman still has her uterus then she needs constant or cyclical progesterone to avoid endometrial hyperplasia or malignancy. In this case the patient has the mirena coil to provide this and so only needs a source of oestrogen as per (A).

(B) is a combination thereapy and so is (E), so they wouldn’t be appropriate.

Vagifem (C) is a topical treatment for atrophic vaginitis and would do nothing for her vasomotor symptoms.

Oestrogen implants (D) are supraphysiological levels of oestrogen and would help with the symptoms but there would be rapid rebound when they ran out.

66
Q

A 19-year-old biochemistry student is seen in your clinic worried about her
hormone levels. She has been told by her GP that her progesterone is low. You enter into a long discussion about the effects of progesterone on the body. Progesterone:

A. Enhances endometrial receptivity
B. Stimulates endometrial growth
C. Increases uterine growth
D. Increases fat deposition
E. Increases bone resorption

A

A. Enhances endometrial receptivity

Endometrial growth (B) occurs prior to ovulation in response to rising oestrogen. progesterone increasing during the follicular stage prepares the uterus for implantation (A).

C,D, and E are all well charecterised effects of oestrogens.

67
Q

A 41-year-old mother of two presents to the GP with long-standing heavy
menstrual bleeding which has become worse over the past year. She is otherwise
well and has no significant medical history. She requests treatment to alleviate
the impact of her heavy bleeding on her social life. Pelvic examination reveals a
normal sized uterus. What is the most appropriate first line treatment?

A. Levonorgestrel-releasing intrauterine system
B. Tranexamic acid
C. Mefenamic acid
D. Tranexamic acid and mefenamic acid combined
E. Vaginal hysterectomy

A

A. Levonorgestrel-releasing intrauterine system

this woman needs symptom control from heavy menstrual bleeding.

The NICE reccomended first line treatment for HMB is a IUS, such as Mirena, this is only in cases without uterine abnormality. Tranexamic and Mefenamic are second line treatments for HMB. Tranexamic acid is usually prescribed alone as it has less of a side effect profile.

A hysterectomy is an option offered to women who have not had resolution from other methods and have completed thier families. Vaginal approach is preferred when there are no complications.

68
Q

A 42-year-old woman is seen in the gynaecology clinic. She has been suffering
from severe premenstrual symptoms all her life. They have now significantly
affected her relationship and her husband is filing for divorce. She comes to your
clinic in tears regarding the future of her children. She demands a hysterectomy
and bilateral salpingoophrectomy. After taking her history you talk about other
less radical treatments. Which management option is inappropriate?

A. Antidepressants
B. Vitamin C
C. Exercise
D. Cognitive behavioural therapy
E. Yasmin – combined oral contraceptive pill

A

B. Vitamin C

Premenstrual syndrome is defined as a condition associated with distressing physical, behavioural and psychological symptoms, but in the abscence of organic or underlying psychiatric disease. It also recurs regularly during the luteal phase of each menstrual cycle, and disappears or significantly regresses at the end of menstruation. First line treatments include: SSRIs (A), Vitimin B6, Improved diet and physical exercise (C), CBT (D), and a trial of Yasmin or Cilest (COCs)(E). There is no evidence to support vitinmin C as a treatment.

Complimentary therapies that have some evidence include; St John’s wart, Ginko Biloba, and Evening Primrose Oil

69
Q

A 22-year-old woman is seen in accident and emergency with lower abdominal
pain and some vaginal discharge. She has had PID once in the past and was treated for it. She is otherwise well. Her temperature is 36.9°c, pulse 90, blood pressure 105/66 mmHg. She is passing good volumes of urine. On clinical examination she has diffuse lower abdominal tenderness. There are no signs of peritonism on examing her abdomen. On vaginal examination she has adnexal tenderness and an offensive discharge. Her CRP is 28 mg/L and her white blood count is 12.2 × 109/L. Her pregnancy test is negative. She is reviewed by your senior and is diagnosed with PID. What would be an appropriate antibiotic regime?

A. IV ceftriaxone and IV doxycycline
B. IV ofloxacin and IV metronidazole
C. IM ceftriaxone, oral doxycycline and oral metronidazole
D. IV clindamycin and gentamicin
E. Oral azithromycin and benzylpeniciilin

A

C. IM ceftriaxone, oral doxycycline and oral metronidazole

This woman is unwell with PID (bilateral adenexal tenderness, abnormal discharge, fever over 38c, PV bleeding, deep dyspareunia and cervical motuion tenderness).

Options A, B and D are all IV antibiotics for use in sever PID with evidence of sepsis, a tubo-ovarian abcess or in pregnancy (obviously avoiding teratogenic tetracyclines).

In this woman’s case she does not have signs of peritonism, fever systemic infection or abscess, so option C is preferred.

Option E is not a recognised treatment for PID in the UK.

70
Q

A 24-year-old woman is in her first pregnancy. She has no significant medical history. She is 40 weeks and 2 days pregnant and has been contracting for 4 days. She is not coping with the pain. She has been given intramuscular pethidine. On examination she is found to be 4 cm dilated (fetus in the occipito-posterior position) having been the same 4 hours previously. What analgesia would you recommend?

A. Remifentanil
B. Pethidine
C. Diamorphine
D. Epidural injection
E. Entonox

A

D. Epidural injection

This woman had a very long latent stage of labour (over 24hrs), she is now in the active phase. This is likely to be a long labour due to the occipto-posterior fetal position.

All the options here are used in labour

Remifentanil (A) is an infusion given where there is a contraindication to a spinal or epidural.

Pethidine (B) has already been given before with little effect so it is unlikely to be acceptable.

Diamorphine (C) is much the same as pethidine and different units use different opiates. Primarily opiates have a sedative efect in labour, rather than an analgesic effect.

Entonox (E) is a good adjunct to pain relief but is unlikey to be enough for the remainder of her labour.

An epidural (D) is the best course of action based on the likely length of the upcoming labour and the likely need for augmentation.

71
Q

A 36-year-old woman is 41 weeks pregnant and is established in spontaneous
labour. She is contracting three times every 10 minutes and has ruptured her
membranes. She is draining significant meconium stained liquor. Her cervix
is 7 cm dilated. Her midwife has started continuous electronic fetal monitoring
using a cardiotocograph (CTG). The baseline rate has been 155, with variability
of 2 beats per minute, for the past 60 minutes. There are no accelerations and no
decelerations. What is the most appropriate management?

A. Pathological CTG – needs delivery
B. Suspicious CTG – needs delivery after fetal blood sampling (FBS)
C. Suspicious CTG – change maternal position, intravenous fluids and
reassess in 20 minutes
D. Suspicious CTG – perform fetal blood sampling and deliver if abnormal
E. Normal CTG – do nothing

A

D. Suspicious CTG – perform fetal blood sampling and deliver if abnormal

A pathological CTG has two or more non-reassuring features or one abnormal feature. A suspicious CTG has one non-reassuring feature. THis woman’s CTG shows a non-reassuring lack of variability and so is suspicious. This rules out answers A and E.

option B is wrong because at this stage we don’t know if delivery is the best option.

Option C would only be correct if we thought there was a fetal blood supply compromise, which given the normal heart rate is unlikely.

72
Q

A 19-year-old woman is giving birth to her first baby. She has been pushing for
an hour and the fetal head has been on the perineum for 6 minutes. There seems
to be a restriction due to resistance of her tissues. Her midwife carries out a right
mediolateral episiotomy. Which of the following structures should not be cut with
the episiotomy?

A. Bulbospongiosus
B. Superficial transverse perineii (STP)
C. Vaginal mucosa
D. Perineal membrane
E. Ischiocavernosus

A

E. Ischiocavernosus

The ischiocavernousus runs from the crus of the clitoris to the ischial tuberosity, the episiotomy should avoid this.

73
Q

A 25-year-old woman in her first pregnancy has a pathological CTG. Her cervix
is 5 cm dilatated. Which of the following will not increase the risk to the fetus if the
doctor performed a fetal blood sample?

A. Human immunodeficiency virus (HIV)
B. Human papilloma virus (HPV)
C. Maternal immune thrombocytopenia
D. Factor IX deficiency
E. Hepatitis C

A

B. Human papilloma virus (HPV)

HIV (A) and Hep C (E) are blood bourne viruses, a FBS will increase the risk of vertical transmission.

(C) and (D) are both potentially also present in the fetus and put it at risk from excessive bleeding.

Only HPV (B) has no risk associated with a FBS.

74
Q

A multiparous woman is admitted to the labour ward with regular painful
contractions. On examination she is 9 cm dilated with intact membranes and is
coping well with labour pains. Forty minutes later her membranes rupture while
she is being examined and you see the umbilical cord hanging from her vagina.
You inform the woman what has happened. She is now fully dilated, the fetal
position is Direct occipitoanterior, and the presenting part is below the ischial
spines. What do you do next?

A. Gain intravenous access, call for help and stop the woman pushing
B. Perform a grade 1 emergency caesarean section
C. Call for help, perform an episiotomy and commence pushing
D. Call for help and prepare for an instrumental delivery
E. Elevate the presenting part by inserting a vaginal pack

A

D. Call for help and prepare for an instrumental delivery

Feeling the cord below the presenting part with ruptered membranes is the definition of cord prolapse, an obstetric emergency.

The most important thing here is to call for help. The woman needs to be taken to theatre and delivered as soon as possible, with the use of instruments likely.

The elevation of the presenting part is important as this will take pressure off the cord. this needs to be done by filing the bladder with saline rather than using packs. (E)

The woman is fully dilated so a caesaren is not indicated (B)

An episiotomy will not be sufficient in this case (C)

Although option (A) is partially correct there needs to be steps towards a definitive plan - the instrument delivery.

75
Q

A 34-year-old para 0 has been admitted for a post-dates induction of labour at
42 weeks. She has received 4 mg PGE2 (prostaglandin) vaginally. After 72 hours
her cervix is 5 cm dilated. Four hours later she is still 5 cm dilated. On abdominal
examination the fetus appears to be a normal size. The fetal head position is left
occipito-transverse, and the station is −1. There is no moulding but a mild caput.
She is contracting two times in every 10 minutes and has an epidural in situ.
You are asked to review and make a management plan. What would be the most
appropriate plan?

A. Re-examine in 4 hours provided the baby is not distressed
B. Discuss the situation with the patient and offer her a caesarean section
C. Start an oxytocin infusion and intermittent monitoring and reassess in
4 hours

D. Insert another 1 mg PGE2 as she is not contracting and reassess in
2 hours
E. Start an oxytocin infusion, commence continuous monitoring and
reassess within an appropriate time span

A

E. Start an oxytocin infusion, commence continuous monitoring and
reassess within an appropriate time span

In this scenario we have a delay in the first stage of labour. Active labour is usualy defined as the point after the cervix is 3cm dilated, as in this case.

This question also highlights the fundamentals of assesing labour, that is, assessing the passage, passenger and power.

This womans labour has arrested (the power) as she is only contracting 2 in 10. The passage has been assessed in the vaginal exam, there is no moulding (change in shape of the head) and a mild caput (swelling of the head).

The abdominal exam indicates that the ‘passenger’ is of normal size.

All of these factors indicate that the problem is with the strength of contractions and action should be taken. Option A isn’t appropriate as its not active management.

The next step here is to offer an augment of labour as oppossed to caeserean (B)

Any labour augmentation will require continous monitoring so (C) is incorrect.

She is now in labour so there is no place for further PGE2 (D)

This makes option E the best answer.

76
Q

A mother comes to labour ward who is low risk, in labour at term. The unit is short
staffed and there are not enough midwives to provide intermittent auscultation of
the fetal heart. You decide to start continuous electronic monitoring (CTG). She
is an epidemiologist and asks you about the CTG and how it will help her labour
and prevent her baby suffering harm. Which of the following would you tell her?
Continuous monitoring has a:

A. High sensitivity and low specificity
B. High sensitivity and high specificity
C. Low sensitivity and low specificity
D. Low sensitivity and high specificity
E. High sensitivity and high positive predictive value

A

A. High sensitivity and low specificity

CTG monitoring was developed to identify cases of fetal acidosis, in reality it’s only evidence is for preventing neonatal seizures. It does, however, have a high false positive rate because of it’s high sensitivity and low specificity (A).

Option B and E would both be very useful tests but there is no such est for this scenario.

Options C and D would both be pretty useless tests. C would basically be only slightly better than guesswork, option D would be accurate but would miss many pathological cases and would lead to deaths.

77
Q

A 29-year-old woman comes to the labour ward complaining that her baby has
not been moving for 72 hours. She is 36 weeks pregnant. Otherwise her pregnancy has been complicated with gestational diabetes for which she is taking insulin. On examination you fail to pick up the fetal heart. You confirm the diagnosis of an intrauterine death. The scan shows no liquor and the baby is transverse. After a long discussion you explain that she unfortunately needs to deliver her baby. What is the best way for her to deliver her baby?

A. Induction with oral mifepristone and oral misoprostol
B. Induction with oral mifepristone and vaginal misoprostol
C. Induction with oral misoprostol
D. Induction with vaginal dinoprostone
E. Caesarean section

A

E. Caesarean section

Diabetes is a risk factor for stillbirth, as in this case.

The usual practice for induction of labour for interuterine death is to use mifepristone to ripen the cerix and misoprostol to induce contractions (A and B). Dinoprostone can also be used for induction of labour (D). Misoprostol alone (C) will not ripen the cervix so is not used alone.

Unfortunately this is not a normal case of induction of labour following a fetal loss. The fetus is lying transverse and with little liquor there is no possibility to manuvour the fetus. Attempting a vaginal delivery in this case has a large risk of uterine rupture or other adverse outcomes. A caesarean is the only option here (E)

78
Q

A 24-year-old woman with gestational diabetes has been progressing normally
through an uncomplicated labour. The midwife delivers the head but it retracts and
does not descend any further. What should the midwife do next?

A. Pull the emergency bell and place the woman in McRobert’s position
B. Place the woman on all fours and instruct her not to push
C. Pull the emergency bell and commence rotational manoeuvres for
shoulder dystocia
D. Pull the emergency cord and ask your helper to apply fundal pressure
E. Pull the emergency bell and prepare for emergency caesarean delivery

A

A. Pull the emergency bell and place the woman in McRobert’s position

This is what is known as ‘turtleneck’ sign, which is when the head delivers and ten apparently retracts. It is an obstetric emergency, and one which a mother with gestational diabetes is particularly prone due to macrosomia.

A series of of manuvours need to be performed in a set order in this scenario. THe most important first step is to place the woman in McRobert’s position (knees and legs maximally flexed) as this gives the pelvic outlet the maximum size.

The majority of cases wil deliver spontaneously in this position.

(B) and (C) are valid manuvours for shoulder dystocia but are not first line. A caesarean is a method used if all otheres have failed.

Applying fundal pressure (D) is definitely a bad idea as this increases the risk of a brachial plexus injury.

79
Q

A 29-year-old multiparous woman is in established labour contracting strongly.
She is 4 cm dilated and had been having regular painful contractions for 6 hours
before they stopped abruptly, heralded by a sudden onset of severe, continuous
lower abdominal pain. The fetal heart trace is difficult to identify, and the
tocometer does not register a signal. What is the most appropriate management?

A. Fetal assessment with formal ultrasound scan
B. FBS
C. Immediate trial of delivery in theatre, with resuscitation facilities on
standby
D. Immediate caesarean delivery
E. Expedite delivery with synthetic oxytocin infusion

A

D. Immediate caesarean delivery

The signs here are very worrying, regular contractions that suddenly stopped, combined with a sever continous abdominal pain are indicitive of uterine rupture. The CTG being unable to detect contractions and a difficult fetal HR to identify are supportive of the diagnosis of rupture, as the uterus is no longer contracting in a organised way.

(A) and (B) are useful tools in suspicious CTG traces, but we are beyond that in this case.

(C) is inappropriate as the woman is nowhere near dilated enough

(E) is not a good choice here as increasing the strength and frequency of contractions will only make matters worse here and may well cause fatal blood loss.

An immediate caesarean is the only sensible management here.

80
Q

A 23-year-old woman is in her first labour. Her cervix is 6 cm dilated and she is in
distress. She is asking for an epidural. Before you call the anaesthetist you check
her history. Which of the following would be an absolute contraindication to an
epidural?

A. Previous spinal surgery
B. Hypotension
C. Mitral stenosis
D. Multiple sclerosis
E. Aortic stenosis

A

B. Hypotension

Hypotension is always a concern with anaestesia, which acts to lower BP further, in the case of an epidural this is an absolute contraindication. Other absolute contraindications include; Patient refusal, allergies to the drugs in question, systemic infection, skin infection over the site, bleeding disorders, platelet count less than 80000/ml, and uncontrolled hypotension.

Conditions that are a relative contraindication include conditions that impede the heart’s ability to compensate; HOCM, Aortic stenosis or mitral stenosis.

Also anotomical abnormalities; spina bifida, or previous spinal surgery, may make things more technically difficult, but are not contraindications.

Condistions like MS may be excacerbated by the epidural, but this is not a contraindication.

81
Q

A 38-year-old nulliparous woman has had an uncomplicated pregnancy. She
has laboured very quickly and is 10 cm dilated. The fetal heart falls to 60 for 4
minutes. She is pushing effectively and the head is 1 cm below the ischial spines.
You prepare for forceps delivery in the room. She has had no analgesia so you
quickly insert a pudendal nerve block and deliver the baby 4 minutes later in good
condition. Which of the following is not a branch of the pudendal nerve?

A. Inferior anal nerve
B. Perineal nerve
C. Dorsal nerve of the clitoris
D. Posterior labial nerve
E. Genital branch of the genitofemoral nerve

A

E. Genital branch of the genitofemoral nerve

If you remember your anatomy the genitofemoral nerve runs through the inguinal canal so is not associated with the pudendal nerve. It arises from L1 and L2.

The first branches of the pudendal nerve are the inferior anal and inferior haemorrhoidal. the other branches listed here also arise from the pudendal nerve.

82
Q

The obstetric team is alerted to a blue-light trauma call expected in accident and
emergency. A 28-year-old woman who is 37 weeks pregnant has been involved
in a high-speed road traffic collision. On arrival, where the obstetric team is on
standby, her Glasgow Coma Scale score is 5 and she has a tachycardic hypotension. What is the most appropriate management sequence?

A. Resuscitation according to Advanced Trauma Life Support (ATLS)
guidelines and transfer to the labour ward
B. Transfer to the CT scanner in preparation for immediate trauma
laparotomy
C. Resuscitation according to ATLS guidelines and fetal assessment with
the patient in left lateral tilt
D. Resuscitation according to ATLS guidelines with immediate caesarean
delivery
E. Resuscitation according to ATLS guidelines and corticosteroids for fetal
lung maturation

A

C. Resuscitation according to ATLS guidelines and fetal assessment with
the patient in left lateral tilt

Transferring the patient in this condition is not a wise move, this rules out (A) and (B).

There is no need for steroids at 37 weeks (E)

Opting for a premorbid caesaren is a serious step, and in this case the woman still has a cardiac output that may well respond well to resuscitation. As such option (D) is not warranted here.

The book answer: Placing the woman in the left lateral positon will help with venous return (Edit: assuming no spinal trauma I hope). Fetal assesment will help with determining a management plan.

83
Q

A 24-year-old woman is seen after her normal vaginal birth. The midwife who
delivered the baby is concerned that there is a third degree tear. Having examined
the woman the obstetrician confirms a third degree tear. The woman is taken to
theatre to repair the external anal sphincter. Which of the following is not a risk
factor for third degree tear?

A. Forceps delivery
B. Second stage of labour lasting over an hour
C. Shoulder dystocia
D. Ventouse delivery
E. Maternal age

A

E. Maternal age

The answer in the book is somewhat garbled. this is the explaination of 3rd degree tears;

1% of all vaginal births result in a 3rd degree tear, these should be identified at delivery and repaired appropriately. a 3a tear invloves less than 50% of the external anal sphincter, a 3b involves more than 50%. A 3c tearinvolves the external and internal anal sphincters.

The use of instruments and ventouse is a risk factor for obvious reasons (A and D)

A long second stage puts a greater mechanical strain on the area so is a risk factor (B)

and shoulder dystocia (C) is always bad news, and will put a much larger strain on the perineum.

That leaves maternal age (E), which according to some published research is also a risk factor for 3rd degree tears. The evidence for this is equivocal though, and so, of a set of answers it is the best.

84
Q

A 34-year-old woman is brought straight to intensive care from the obstetric
theatre after an emergency caesarean section for fetal distress. The attending
obstetrician remarks that she is showing haematological signs of disseminated
intravascular coagulation. Which blood profile is she most likely to have?

Prothrombin time (PT) ↓ ↓ ↑ ↑ →
Activated partial thromboplastin time
(aPTT) → → ↑ ↑ ↑
Bleeding time ↑ ↑ ↑ ↑ ↑
Platelets → ↑ ↓ ↑ →
Active haemorrhage Yes No Yes No No

A B C D E

A

C - Increased PT, Incresed aPTT, increased bleeding time, Reduced Platelets, Active haemorrhage.

If you remember that DIC is a consumptive coagulopathy then this is a straightforward question. Because DIC activates teh intrinsic and extrinsic pathways and consumes the materials involved it leads to an increase in BOTH the aPTT and PT. Straight away there are only two possible answers (C and D)

Only (C) includes the decreased platelet count found in DIC, in fact it is the only answer of the 5 that does, so only knowing that DIC involves reduced platelets gets you the answer.

Of note, answers A and B don’t really make sense and E describes a congenital condition such as von Willebrand’s, which reduces platelet adhesion.

85
Q

A 31-year-old undergoes a planned caesarean section for a breech presentation.
After delivery of her healthy baby there is difficulty in delivering the placenta, as it
is adhered to the uterus. She has lost 5 L of blood as a result of the placenta accreta. The placenta has been removed but she is still bleeding and is cardiovascularly unstable despite blood product replacement. What would be the most management to definitively arrest haemorrhage?

A. Syntocinon infusion
B. B-Lynch suture
C. Internal artery ligation
D. Hysterectomy
E. Intrauterine balloon

A

D. Hysterectomy

This woman is in serious trouble, the placenta acreta has led to her losing 5l of blood (effectively her entire circulating volume).

Syntocinon (A) helps the uterus contract which acts to shut off the blood vessels in the uterus.

A B-lynch suture (B) is an external uterine suture which helps to contract the uterus.

Internal illiac [sic] artery ligation (C) would reduce blood flow to the uterus, but there is still a suply from the ovarian arteries.

An intrauterine balloon (E) acts to tamponade the blood flow, much like the surgeon’s hands, the so-called factor XIV…

All of the above could have formed part of the management plan earlier in this case, but teh extreme blood loss means that only one plan could result in saving this woman’s life.

86
Q

A 39-year-old woman is 6 days post-partum and has come back to hospital with
shortness of breath. She is struggling to breath at rest, has a respiratory rate of
28, pulse 115, BP 105/60 mmHg, temperature 37.4° C. On examination she has an
audible wheeze and cough. Investigations reveal a PO2 of 9.5 kPa on arterial blood gas and a PCO2 3.7 kPa, pH 7.36, base excess -3.4. A chest x-ray shows some upper lobe diversion and bilateral diffuse shadowing with an enlarged heart. Her haemoglobin is 8.9 g/dL, white blood count 11.1 × 109/L and C-reactive protein 21 mg/L. What is the most likely cause of her symptoms?

A. Lower respiratory tract infection
B. Pulmonary embolism
C. Peri-partum cardiomyopathy
D. Systemic inflammatory response syndrome (SIRS)
E. Post-partum anaemia

A

C. Peri-partum cardiomyopathy

This woman’s tachycardia, tachypnoea, low grade pyrexia and cough are suggestive of a LRTI (A) but the hypoxia would point towards a PE type picture (B). The abscence of chest pain and respiratory alkolosis goes against this.

She does not meet the diagnostic criteria for SIRS (D) as there is no proof of infection.

Her Hb is low but not low enough to be expected to be symptomatic. (E)

This leaves a peri-partum cardiomyopathy (C), a diagnosis of exclusion. It is a rare condition associated with; multiple pregnancy, advanced maternal age, and hypertension in pregnancy. It has hallmarks of congestive cardiac failure. It is most common in th elast month of pregnancy and up to 5 moths post partum.

87
Q

A 17-year-old girl is seen in accident and emergency 14 days after an emergency
caesarean delivery of a healthy infant, her first. Her neighbours became concerned and called the police. She had been seen prostrate in the garden chanting verses from the Bible and shouted at them accusing them of being spies when they asked if she was ok. They say her problem has worsened over the past fortnight. What is the most likely diagnosis?

A. Post-partum depression
B. Bipolar affective disorder
C. Puerperal psychosis
D. Schizophrenia
E. Acute confusional state (delirium)

A

C. Puerperal psychosis

This woman is clearly psychotic, and the cause can be determined by looking at the options here.

Psychotic symptoms in sever depression will be nihilistic rather than paranoid, this makes (A) unlikely.

There is no history of Bipolar affective disorder (B) here, it is a long standing diagnosis, not 2 weeks. Psychosis in mania also tends to be delusional. There is also no mention of depressive symptoms.

Schizophrenia (D), by definition has to be enduring over 6 months.

An acute confusional state (E) is a fluctuating mental state due to an underlying condition such as an infection. There is nothing here to sugesst it’s fluctuating and no mention of infection.

This has to be a Puerperal psychosis (C), AKA a post partum psychosis.

risk factors include caesaren section, emergeny delivery, and primiparity (all present here)

88
Q

At birth, which of the following does not occur in the fetal circulation?

A. Right ventricular output increases
B. A decrease in venous return
C. Closure of the foramen ovale
D. Pulmonary artery vasoconstriction
E. Closure of the ductus arteriosus

A

D. Pulmonary artery vasoconstriction

This quesion is poorly written, I selected option D straight away without much thought. THis is because the question says ‘at birth’. the ductus (E) closes over the course of the first couple of days of life, conversely the foramen ovale closes on the first breath after birth.

Anyway, the best answer is (D) as it is absolutely incorrect. the pulmonary arteries vasodilate to allow for the lower pressure in the right sided circulation after birth.

The occlusion of the umbilical vessels lowers venous return (B), the expansion of the lungs with air lowers the pulmonary circulatory pressure. These act to change the pressure differential across the foramen ovale, closing it (C). The right ventricle increases it’s output due to lower resistance (A)

89
Q

A woman on the labour ward has just had a normal birth. At birth there was
a lot of meconium present. The newborn did not respond initially but did after
subsequent resuscitation. The midwife records the Apgar score as 5. Which of the
following best describes the categories an Apgar score is created from?

A. Tone, colour, noise, pulse and blood pressure
B. Tone, colour, respiratory effort, heart rate and reflex irritability
C. Tone, colour, pulse, reflex irritability and blood pressure
D. Tone, colour, pulse, respiratory effort and blood pressure
E. Tone, colour, cry, blood pressure and heart rate

A

B. Tone, colour, respiratory effort, heart rate and reflex irritability

Tone and colour are in all the answers, we just need to decide which are the other three measurements. One easy way is that with some time spent on neonates you will know that blood pressure isn’t measured in newborns…

Even without that knowledge onlt two of these chices (E and B) refer, correctly, to heart rate, the others all say pulse. and of these two, (E) makes no effort to check respiratory effort.

(B) is our answer.

90
Q

An 18-year-old woman has been successfully delivered of a healthy female
infant by elective caesarean section for maternal request. Estimated blood loss
was 1120 mL. Forty minutes after return to the recovery area, she has a brisk
vaginal bleed of around a litre. Her pulse rate is 120 bpm and blood pressure is
95/55 mmHg. What should the immediate management process be?

A. Rapid fluid resucitation, uterine massage, intravenous ergometrine
B. Rapid fluid resuscitation, intravenous ergometrine and bimanual
compression of the uterus
C. Rapid fluid resuscitation, insertion of an intrauterine balloon catheter
device
D. Rapid fluid resuscitation, uterine massage, oxytocin infusion and
vaginal assessment
E. Rapid fluid resuscitation and administration of direct
intramyometrial uterotonic agents

A

D. Rapid fluid resuscitation, uterine massage, oxytocin infusion and
vaginal assessmentIt is important here to recognise that this is a primary post-partum haemorrhage. All of the options listed are recognised steps in the management of primary PPH, but only (D) is the first line. In the first instance a priamry PPH should be managed by fluid resuscitation, uterine massage and oxytocin.

IV ergometrine (A) and bimanual compression (B) are second line steps if initial steps have failed to control the bleeding.

A balloon catheter (C) and intramyometrial uterotonic agents (E) are only used if other interventions have failed.

The other help in answering this question is that only option (D) includes a vaginal assesment, always a sensible thing to do in the first instance in a PV bleed!

91
Q

A 34-year-old woman develops a significant post-partum haemorrhage and
hypotensive shock following vaginal delivery of a healthy infant at term. The
labour was uncomplicated. She recovers well with volume replacement and
oxytocin and returns to the post-natal ward. She is unable to breast feed on the
ward and 2 months later has neither started breastfeeding nor resumed her periods and is increasingly fatigued. What is the most likely diagnosis?

A. Addison’s disease
B. Syndrome of inappropriate antidiuretic hormone hypersecretion (SIADH)
C. Sheehan’s syndrome
D. Panhyperpituitarism
E. Post-partum depression

A

C. Sheehan’s syndrome

The inability to lactate (in a motivated mother) coupled with the lack of periods points towards hypopituatarism as there appears to be a deficit in prolactin and FSH/LH. This is clearly not a hyperpituitarism (D)

Depression (E) doesn’t seem evident here and it doesn’t explain the continued abscence of periods.

(A) and (B) are not in line with this case.

Sheehan’s is an ischemia of the pituatary that can occur after a significant bleed in pregnancy, which causes a loss of perfusion to the hyperplastic pituitary.

92
Q

A 30-year-old French woman delivers a live female infant by spontaneous vaginal
delivery at term. In the eleventh week of pregnancy she developed a flu-like
illness which resolved spontaneously a week later. Her newborn child has severe
hydrocephalus and chorioretinitis. Four days after birth, she develops severe
convulsions and efforts to revive her are unsuccessful. Which pathogen is most
likely to be responsible?

A. Cytomegalovirus (CMV)
B. Human immunodeficiency virus
C. Toxoplasma gondii
D. Group B Streptococcus
E. Listeria monocytogenes

A

C. Toxoplasma gondii

CMV (A) is very common and appears as a mild flu like illness, complications for the fetus include; visual and hearing loss, microcephaly and long-term neurodevelopmental disability. Babies will not have hydrocephalus though, making this the wrong answer.

HIV (B) should have been screened for, but it could present as a flu-like illness in seroconversion. Once again, the symptoms in the infant do not correlate with HIV.

Strep (D) would not cause the hydocephalus

Listeria (E) is classically linked with unpasturised cheese and pate, it causes misscarridge, stillbirth and early delivery.

The correct answer here is Toxoplasmosis (C), classically associated with undercooked meat and cat faeces. Initial maternal infection is mild but it causes sever sequele in the infant.

93
Q

A 32-year-old woman has a routine cervical smear at her GP practice. The result
returns as severe dyskaryosis. Following colposcopy and cervical biopsy, formal
histological examination reveals cervical intraepithelial neoplasia 3 (CIN 3). Which
of the following pathogens is the most likely to have caused this disease?

A. Candida albicans
B. Human immunodeficiency virus (HIV)
C. Human papilloma virus (HPV)
D. Herpes simplex
E. Treponema pallidum

A

C. Human papilloma virus (HPV)

you should know this!

94
Q

A 15-year-old girl attends the paediatric gynaecology clinic with primary
amenorrhoea and features of secondary breast development. She has intermittent
abdominal bloating and is extremely worried that she is ‘not like other girls’. On
speculum examination of the vagina, which is normal externally, a bulging red
disc is seen 3 cm proximal to the introitus. What is the most likely diagnosis?

A. Turner’s syndrome
B. Congenital adrenal hyperplasia
C. Imperforate hymen
D. Anorexia nervosa
E. Delayed puberty

A

C. Imperforate hymen

primary amenorrhoea is a failure to start periods, it can be due to a structural abnormality or a problem with the hypothalamic-pituitary axis (HPA).

This girl’s normal external genitalia and breast development indicate that the HPA is funtioning correctly. This rules out (E), (B), and (A)

Anorexia (D) would cause a lack of secondary development if it started before puberty or secondary amenorrhoea, neither is the case here.

In this case the problem is that the hymen has failed to seperate at the time of menarche and this has led to a build up of menstrual blod in the pelvis/abdomen. This is known as a haematocolpos and is what is seen in this case as a bulging red disc.

95
Q

A 19-year-old woman undergoes surgical evacuation of the retained products
of conception (ERPC). Histological examination of the sample shows genetically
abnormal placenta with a mixture of large and small villi with scalloped outlines,
trophoblastic hyperplasia. What is the most likely diagnosis?

A. Choriocarcinoma
B. Degenerated uterine leimyoma
C. Uterine dysgerminoma
D. Hydatidiform mole
E. Complete miscarriage

A

D. Hydatidiform mole

The pathological findings here are consistent with a hydatidiform mole. This is part of specrum of benign trophoblastic tumours found in pregnancy. The usual presentation is of unusual or heavy bleeding after the 6th week of pregnancy with a pregnancy that is larger than dates suggest. Pre-eclampsia, thyrotoxicosis and hyperemesis Gravidarum are common also.

bhCG assesment clinically will reveal a massively raised leve, and the ultrasound will show a ‘snowstorm’ appearance. If the bhCG levels don’t fall during serial measurement then further action is needed.

Choriocarcinoma (A) is an agressive and malignant form of trophoblastic disease, it has histological findings of closely related syccytiotrophoblasts and cytotrophoblasts, there will also be syncytiotrophoblasts with abnormal nuclei and high eosiniphil counts.

(B) would not show histology related to pregnancy.

This can’t be a complete miscarriage (E) as the woman is undergoing a ERPC.

Uterine dysgerminomas (C) are distinct germ cell tumours which will have lobular cells with fibrous stromal cells and lymphocytic invasion.

96
Q

An 89-year-old woman attends the gynaecology clinic with a long history of a dragging sensation in the vagina. Apart from severe aortic stenosis, she has no significant medical history. She leaks fluid when she sneezes or coughs. On examination with a Sims’ speculum in the left lateral position, a grade 1 uterine prolapse is seen, with an additional cystocoele. What is the most appropriate management?

A. Vaginal hysterectomy with anterior colporrhapy (cystocoele repair)
B. Vaginal hysterectomy alone
C. Tension-free vaginal tape (TVT)
D. Weight loss and pelvic floor exercises
E. Twice weekly 0.1 per cent estriol cream and insertion of shelf pessary

A

E. Twice weekly 0.1 per cent estriol cream and insertion of shelf pessary

This woman has a bladder prolapse that is causing symptoms so treatment is needed. Unfortunately her age and severe aortic stenosis leave her unfit for surgery, this leaves just options (E) and (D).

Frankly option (D) is a bit ridiculous in an 89 year-old

97
Q

A 46-year-old woman presents to your clinic with a 6-year history of Incontinence.
She has had four children by vaginal deliveries, has a body mass index (BMI)
of 35 kg/m2 and suffers from hayfever. Initial examination reveals a very small
cystocele. A mid-stream urine culture is negative and urodynamic studies show a
weakened urethral sphincter. What is the most appropriate first line management?

A. Fesoterodine 4 mg daily
B. Weight loss and pelvic physiotherapy
C. Tension free vaginal tape
D. Solifenacin 5 mg daily and pelvic physiotherapy
E. Anterior repair and insertion of a transobturator tape

A

B. Weight loss and pelvic physiotherapy

with the woman’s BMI given as 35 in the question, weight loss is always going to be the sensisble first step.

Fesoterodine (A) and Solifenacin (D) reduce bladder activity to help with urge incontinence.

It’s not clear but it’s likely this is stress incontinence.

(C) and (E) are both god surgical choices but in the first case (B) is definitely the choice here.

98
Q

A 16-year-old girl attends the gynaecology clinic complaining of vaginal itching
and lumpy labia. On examination the area is covered with vulval warts. Which is
the causative pathogen for vulval warts?

A. Human papilloma virus type 16
B. Human papilloma virus type 18
C. Human papilloma virus type 6
D. Herpes simplex virus
E. Epidermophyton floccosum

A

C. Human papilloma virus type 6

HPV a6 and 18 (A) and (B) are associated with CIN not warts

HSV (D) causes a different infection entirely, papulovesicular morpholog is the clasical description.

Epidermophyton floccosum (E) is a fungal infection that presents as tinea cruris.

HPV 6 (C) and 11 are asscoiated with genital warts.

99
Q

A 25-year-old woman attends accident and emergency with an exquisitely sore,
large swelling of her vagina which she noticed only a couple of days before.
It has steadily got much bigger. On examination there is a soft fluctuant mass
on the right labia minora which is very tender. What is the most appropriate
management?

A. Marsupialization
B. Oral ofloxacin and metronidazole
C. Sebaceous cystectomy
D. Local 2 per cent clotrimazole (Canestan)
E. Referral to a vulval clinic

A

A. Marsupialization

The history strongly suggests a Bartholin’s abscess, the bartholin’s glands lie in the labia minora at the 8 and 4 o’clock positions and are normally non-palpable. If the ducts become blocked then a situation as in this case can develop.

The treatment is to remove the top of the abcess and suture the edges down, it will then shrink down as it drains and this prevents recurrence. This is marsupilization (A).

antibiotics (B) would be insufficient at this late stage, additionally this regime is for PID.

This isn’t a sebaceous cyst (C)

And this is also not thrush (D)

No specialist assesment is needed here (E), you would refer if there was concerns that this mght be malignant.

100
Q

An 18-year-old woman attends clinic seeking contraceptive advice. She is
currently using condoms only and is keen to start taking the combined oral
contraceptive pill (COCP). Her sister used to take it but told her there were lots of
problems with it. Her aunt has bowel cancer and she has no other past medical
history. Appropriate counselling should cover all of the following except:

A. There is an overall 12 per cent reduction in risk of cancers

B. There is a small increase in cervical cancer with prolonged use
(>8 years)
C. There is a reduction in the risk of bowel cancer
D. There is an increase in the risk of ovarian cancer
E. There is no need for a cervical smear prior to starting the pill

A

D. There is an increase in the risk of ovarian cancer

The COCP is associated with a decresed risk of cancer overall (A) and a specific decrease in risk of bowel and rectal cancer (C)

As this woman is under 25 there is no need for a smear test (E), the small increased risk of cervical cancer (B) with long term use is mitigated by the screening program that starts at 25.

The use of a COCP is actually associated with a reduction in the risk of ovarian cancer (D)

101
Q

A 49-year-old woman presents to a private clinic expressing her desire to become
pregnant. She has no past medical history. Initial investigations show that she still
has ovarian function, is ovulating and is having regular periods. An ultrasound
of her pelvis shows no structural abnormality and an hysterosalpingography
demonstrates patent fallopian tubes. Analysis of her partner’s semen is normal.
Which would not be an appropriate first line management option?

A. In vitro fertilisation (IVF)
B. Intracytoplasmic sperm implantation
C. Intrauterine insemination
D. Clomiphene
E. Egg donation IVF

A

D. Clomiphene

Given the low chances of sucess of IVF in this age group any of options (A,B,C, and E) would be appropriate. In fact (E) might offer the best chance of sucess as the donor eggs may well be of better quality.

Clomiphene is a selective oestrogen receptor modulator and would be used to induce ovulation. In this case the woman is already ovulating so it is unecessary.

102
Q

A 42-year-old woman presents to the urogynaecology clinic with a 3-year history of urge incontinence. She has features of an overactive bladder and is desperate to start treatment for her problem as it is affecting her quality of life. She opts for medical treatment. What is the most appropriate first line pharmacological therapeutic?

A. Darifenacin
B. Oxybutynin
C. Fesoterodine
D. Solifenacin
E. Oxybutynin dermal patch

A

B. Oxybutynin

Diagnosis of an overactive bladder is made clinically, urodynamic studies can confirm but this is an invasive step. In is usually appropriate to trial a medical therapy such as the options here.

Oxybutynin (B), as an immediate release, is the first line treatment for overactive bladder. Alkl the other options are second line treatments.

It is improtant to counsel this patient on anti-muscarinic side effects - dry mouth, constipation, and urinary retention.

103
Q

A 41-year-old woman is about to undergo her first cycle of IVF. As part of the
consultation, she is counselled about the maternal and fetal risks involved with
IVF-conceived pregnancies. All of the following occur in such pregnancies except:

A. Increased risk of low birth weight infants
B. Increased risk of fetal congenital abnormalities
C. Decreased risk of ectopic pregnancies
D. Increased risk of small for gestational age (SGA) fetuses in singleton
pregnancies
E. Increased risk of maternal pregnancy-induced hypertension (PIH)

A

C. Decreased risk of ectopic pregnancies

There are many risks associated with IVF conception, (C) is incorrect as there is actually an increased risk of ectopics in IVF.

The use of a donor egg is associated with a 7-fold increase in PIH (E) but the mechanism is unknown.

104
Q

16-year-old presents to the termination of pregnancy service 6 weeks into her
second pregnancy requesting surgical termination (STOP). What is not required as
part of her work-up for the procedure?

A. Antibiotic prophylaxis for Chlamydia
B. Gaining consent from her mother
C. Contraception discussion
D. Explaining the risks of STOP
E. Explaining that the risk of uterine perforation is one in 300

A

B. Gaining consent from her mother

This one is relatively straightforward as long as you don’t overlook the relevant fact that she is 16 years old. As she is 16 she is pressumed Gillick competent and so can consent to any procedure offered without need to involve a parent (B).

All the other options are required components of a well managed STOP.

105
Q

A 35-year-old woman is seen in the assisted conception unit. She has been trying
to conceive for 4 years. In this period she has been having regular intercourse. Her
periods have been irregular and recently she has had no periods at all. Her BMI is
19.5 kg/m2, she has had an appendectomy and is otherwise well. Her biochemistry
comes back as follows: luteinizing hormone (LH) 0.5 IU/L, follicle-stimulating
hormone (FSH) 1.0 IU/L, prolactin 490 mIU/L, thyroxine (T4) 12, thyroid stimulating
hormone (TSH) 4.2 mIU/L, oestradiol 60 pmol/L. What is the most likely cause of
her subfertility?

A. Polycystic ovarian syndrome (PCOS)
B. Hypothyroidism
C. Microprolactinoma
D. Hypothalamic hypogonadism
E. Anorexia

A

D. Hypothalamic hypogonadism

in this scenario we see reduced levels of FSH and LH, so much so that her ovaries are not producing oestradiol. But tellingly because the LH and FSH are low it means that the negative feedback loop has not worked, there is a hypothalamic issue here (D)

Her BMI is just below normal so not pathological (E)

Prolactin is also normal (C)

TSH is high normal so not significant (B)

She likely fulfils the oligo/anovulation criteria of PCOS (A) but there is no information on the hyperandrogenism or polycystic ovaries critera.

106
Q

A 19-year-old comes to you for some pre-conception advice. Some members of
her family and her partner’s family have a sickle cell anaemia. She reveals that her
sister and his sister are both affected. Tests have shown that they are both carriers. What is the chance that if their child was a boy he would have sickle cell anaemia?

A. 50 per cent
B. 67 per cent
C. 100 per cent
D. 33 per cent
E. 25 per cent

A

E. 25 per cent

Sickle cell anaemia is an autosomal dominant condition, so the gender of teh offspring and the family history is largely irrelevant. The fact that both parents are carriers puts the risk at 25% of the offspring being afected by the disease (E).

There is a 50% chance of the offspring being sickle cell trait though.

107
Q

An 18-year-old girl is seen in the colposcopy clinic after having had persistent
post-coital bleeding. She has been sexually active since the age of 14 and has no
past medical history. She is studying for her A-levels and has been doing a lot
of reading. She is concerned that she might have cervical cancer. Which of the
following is not a risk factor for cervical cancer?

A. Herpes simplex virus (HSV)
B. Smoking
C. HIV
D. Use of the oral contraceptive pill
E. Multiparity

A

A. Herpes simplex virus (HSV)

Human papilloma virus is associated with cervical cancer but not HSV (A)

Smoking is a risk factor for pretty much anything (B)

Any immunocompromise increses the chance of a CIN developing into a malignant type, HIV is an example of this (C)

COCP (D) and multiparity (E) are both linked factors because of the risky behaviour associated with it.

108
Q

A 49-year-old comes to the urogynaecology clinic with a history of leaking
urine for the last year. There are associated stress symptoms and some urge
symptoms. Interestingly she says that it seems to come from inside the vagina
as well. She had a hysterectomy last year for endometrial cancer and had
quite a prolonged recovery. She has a BMI of 30 kg/m2, does not smoke and
is otherwise fit and well. You are suspicious that she might have a vesicovaginal
fistula secondary to her operation. What is the most appropriate first
line investigation?

A. Examination under anaesthesia (EUA) and cystoscopy
B. Pelvic MRI
C. Instillation of methylene blue into the urinary bladder and speculum
examination
D. Pelvic computed tomography
E. Urodynamic study

A

C. Instillation of methylene blue into the urinary bladder and speculum

This is an appropriate first line investigation that can be carried out in the clinic. By passing the dye into the bladder the woman can be examined to determine where the dye is emerging.

EUA and cystoscopy (A) would be an appropriate second line investigation.

Urodynamics (E) wouldn’t help identify stress incontinence, equally imaging (B and D) won’t help to visualise the structures in question with any accuracy.

109
Q

A 16-year-old girl presents to your surgery with a history of unprotected sexual
intercourse (UPSI) 70 hours ago. Her last menstrual period was 8 days ago. Her
only past medical history of note is that of epilepsy which is well controlled
by carbamazepine. She is worried about becoming pregnant, does not want her
mother to find out and is in a hurry to get home before suspicions are raised.
Which of the following options are available to her?

A. Take the combined oral contraceptive pill (COCP) continuously for the
next month
B. A copper intrauterine device (IUD) should be inserted with prior
screening for sexually transmitted infections (STIs)
C. Levonorgestrel 1.5 mg should be given as she is within 72 hours of
UPSI
D. Reassure and tell her to come back when she has made her mind up as
ulipristal can be taken up to 7 days after UPSI
E. Reassure her that she is in the safe part of her cycle and she should try
and use condoms in the future

A

B. A copper intrauterine device (IUD) should be inserted with prior
screening for sexually transmitted infections (STIs)

Option (A) would not be apropriate as an emergency contraception and she is on an enzyme inducer.

(E) is incorrect advice, she may well be at risk.

Realistically she could be offered the copper coil (B), which is effective up to 5 days after the UPSI, the progesterone antagonist ulipristal (D) also effective up to 5 days after UPSI, and Levonorgestrel (C) effective up to 72 hours after the UPSI.

The fact that she is on an enzyme inducer reduces thr efficacy of levonorgestrel making it uncertain to be effective. The fact that she is worried about pregnancy and her young age makes the copper coil an advisable option here (B)

110
Q

A 40-year-old woman comes to your clinic alone wanting an effective form of
contraception. She has two children from a previous marriage and has recently
started a new relationship. She says that she does not want any further children.
She has regular heavy periods, no menopausal symptoms and she is otherwise well with no past medical history. A recent ultrasound showed a normal sized uterus and pipelle biopsy revealed normal secretory endometrial tissue. What is the most appropriate form of contraception?

A. Combined oral contraceptive pill with <30 μg of oestrogen
B. Mirena coil
C. Laparoscopic sterilization
D. Vasectomy
E. Total abdominal hysterectomy

A

B. Mirena coil

The mention of heavy menstruation and the specified normal uterine structure indicates what line of management the question is trying to direct you towards. The mirena will reduce the severity of her periods and may stop them completely, she is to be warned that there may be up to 6 months of spotting.

You can’t reccomend a vasectomy (D) to this patient without consulting the partner.

A total abdominal hysterectomy (E) is a seriously over the top move for a basically healthy woman. Also the abdominal approach for this is bizzare.

Laproscopic sterilisation (C) is a good choice but only if she is sure that she doesn’t want children. the fact that she has just started a new relationship would caution against this option.

The COCP (A) is a good choice also, but it wouldn’t have the added benefit of reducing the severity of her periods.

111
Q

In a busy gynaecology clinic you are assessing a 22-year-old woman who has not
had a period for 18 months. She is not pregnant and previously had regular periods. She has had two surgical terminations of pregnancies (STOP), an underactive thyroid gland and an appendectomy. Clinical examination is unremarkable with a BMI kg/m2 of 20. Biochemical investigations reveal a T4 of 17 pmol/L, TSH 4.6 kg/m2, prolactin of 570 mU/L, and testosterone of 42 ng/dL. LH and FSH are normal. Vaginal ultrasound shows a normal sized uterus and the left ovary contain four cysts. Which of the answers listed below is the most likely cause?

A. Polycystic ovarian syndrome (PCOS)
B. Prolactinoma
C. Sheehan’s syndrome
D. Asherman’s syndrome
E. Anorexia nervosa

A

D. Asherman’s syndrome

PCOS (A) is charecterised by biochemical or physical evidence of hyperandrogenism and an ultrasound scan showing 12 or more follicles in the ovary. There is no evidence of this in this case.

Her prolactin is high, but normal, this makes a prolactinoma (B) unlikey.

Her normal BMI and normal LH and FSH go against anorexia (E)

Sheehan’s (C) in a pituatary failure caused by a post-partumn ischemia/bleed in the pituitary. THere is nothing in the case suggestive that she has just given birth or any sign of hormonal irregularities.

This leaves the likely diagnosis of Asherman’s syndrome (D) a rare complication after uterin surgical procedures where uterine scar tissue forms adhesions. In light of her two previos STOPs this needs to be investigated by hysteroscopy.

112
Q

A 26-year-old woman is otherwise fit and well has been trying to conceive for
over 2 years. On questioning she has regular periods and has been having regular
intercourse. There are no abnormalities on clinical examination. What would be
your first line investigations for her subfertility?

A. Day 14 FSH and LH, ultrasound and hysterosalpingogram (HSG), semen
analysis
B. Day 1–3 FSH and LH, mid-luteal progesterone, semen analysis
C. Day 1–3 FSH and LH, mid-follicular progesterone, semen analysis
D. Random LH, FSH, HSG, semen analysis
E. Ultrasound, laparoscopy, semen analysis

A

B. Day 1–3 FSH and LH, mid-luteal progesterone, semen analysis

A laproscope (E) is never going to be a first line investigation for subfertility

Day 14 FSH and LH (A) would not be helpful as there will be a LH surge at the time of ovulation. In addition a hysterosalpingogram is rather invasive as a first line investigation.

Random hormone levels (D) are a pointless as the stage in the cycle needs to be known.

This leaves us options (B) and (C). of the two (B) is the option of choice as the mid-follicular progesterone levels will always be low. After carrying out a mid-luteal progesterone, a level greater than 30 nmol/L would lead you to determine that the woman was indeed ovulating.

113
Q

A 42-year-old man undergoes semen analysis as part of the investigation of
subfertility with his wife. What result would most likely contribute to their
subfertility?

A. Sperm count 30 million/mL
B. Volume 2.5 mL
C. 40 per cent have normal motility
D. 5 per cent normal morphology
E. pH 7.4

A

C. 40 per cent have normal motility

male factor subfertility is indicated in 25-40% of subfertile couples, Criteria for normal ejaculate are as follows;

Sperm Count should be over 15million per ml (A)

Volume should be between 1.5-6ml (B)

50% normal motility (C)

Over 4% have normal morphology (D)

pH should be between 7.2 and 8.0 (E)

It can be seen from these criteria that the abnormal measurement here is the motility of the sperm, answer (C).

114
Q

A 46-year-old women in her fifth IVF cycle is admitted to the emergency
department 4 days after egg collection. She is complaining of a swollen
abdomen and shortness of breath. She is reviewed and a diagnosis of ovarian
hyperstimulation syndrome (OHSS) is made. Which of the following is not a
clinical feature/complication of OHSS?

A. Hydrothorax
B. Deep vein thrombosis
C. Haemodilution
D. Oliguria
E. Marked ascites

A

C. Haemodilution

OHSS is caused by overstimulatin of the ovaries during IVF, it occurs in a mild form in 33% of all IVF cycles, and as a severe form in 3-8%.

It presents as abdominal pain and swelling, and vomiting. Ovarian torsion, appendicitis and ectopic pregnancy should be excluded

There is the finding of haemoconcentration with hypoproteinaemia and ascites (E), which may lead to pleural effusion (A).

This is why option (C) is incorrect.

This haemoconcentration is why there is an increased risk of DVT (B)

Severe OHSS cuases large third space losses which leads to reduced renal perfusion and oligouria (D)

115
Q

A 17-year-old girl comes to clinic with her mother as she has not started having
periods yet and they are worried. On examination she is of short stature, with a
slightly widened neck and has no secondary sexual characteristics and there is no
obvious abnormality of the external genitalia. What is the most likely diagnosis
form this limited information?

A. Androgen insensitivity syndrome
B. Turner’s syndrome
C. Congenital adrenal hyperplasia
D. Kallmann’s syndrome
E. Rokitansky’s syndrome

A

B. Turner’s syndrome

Lack of secondary sexual characteristics, short stature and a widened (or webbed) neck are clasic features of Turner’s syndrome (B).

AIS (A) would present as primary amenorrhoea but without the syndromic features here

CAH (C) would present with virilisation of the female genitallia and hyperandrogenism.

Kallmann’s (D) could fit here, there is a reduction in gnRH leading to hypogonadism, delayed puberty and a lack of secondary characteristics.

Rokitansky’s (E) is mullerian agenesis, the girl would be 46xx with normal ovaries and so normal secondary characteristics. THere would, however, be agenesis of the uterus, fallopian tubes or proximal vagina.

116
Q

A 22-year-old woman presents to the GUM clinic with an offensive smelling
discharge. She is sexually active and is in a monogamous relationship. She describes no pain or soreness just an offensive smelling discharge. After examination and taking swabs for the second time she is diagnosed with bacterial vaginosis. Which of the following organisms is not likely to be the cause?

A. Gardnerella species
B. Mobiluncus
C. Bacteroides
D. Trichomonas
E. Mycoplasma

A

D. Trichomonas

Trichomonas (D) is a sexualy transmitted infection, which BV is not, it also classically presents with a frothy, foul, green, discharge.

Bacterial vaginosis is a disturbance in the usual bacterial make up of the vagina and is accompianed by a fishy white discharge. aside from Trichomonas, all of the others are bacteria involved in BV.

117
Q

A 28-year-old woman attends her GP clinic for routine cervical screening. Liquidbased cytology (LBC) shows mild dyskaryosis. A repeat sample again shows mild dyskaryosis and positive for Human papiloma virus (HPV). What is the most appropriate management?

A. Repeat the LBC smear test in 6 months
B. Repeat the LBC smear test in 3 months
C. Arrange colposcopy at the gynaecology clinic
D. Knife cone biopsy of the cervix
E. Large loop excision of the transformation zone

A

C. Arrange colposcopy at the gynaecology clinic

I have edited the question to include reference to a HPV co-infection, this makes it work as a question.

Answer (C) is correct as per the NICE guidelines Feb 2015 (copied below)

Ensure the woman has received an explanation as to what an abnormal result means and what further tests or treatment she may need.

Patient information may be found online.

Women with borderline or mild (low-grade) dyskaryosis — the laboratory will test the cervical cytology sample for human papillomavirus (HPV):

If HPV is found the woman will be referred for colposcopy (within 8 weeks).
If HPV is not found the woman will be returned to the routine screening programme (every 3-5 years depending on her age).

Women with moderate or high-grade dyskaryosis will be referred for colposcopy within 4 weeks.
Women with suspected invasive cancer or glandular neoplasia will be referred for colposcpy within 2 weeks.

118
Q

When assessing the fetal presenting part in labour it is important to know the
anatomy of the pelvis. What are the bony landmarks of the pelvic outlet?

A. Pubic arch, ischial tuberosities and the coccyx
B. Pectineal line, ischial spines, coccyx
C. Pubic symphysis, pubic rami, sacrum
D. Pectineal line, ischial tuberosities and the coccyx
E. Pubic arch, ischial spines, sacrum

A

A. Pubic arch, ischial tuberosities and the coccyx

The pectineal line is a groove that runs along the pelvis and isn’t a vorder of the outlet. This makes (D) and (B) incorrect.

The sacrum forms the pelvin inlet not the outlet, so (C) and (E) are wrong.

The outlet can be seen in the attached illustration.

119
Q

A 26 year old undergoes potassium-titanyl-phosphate (KTP) laser laparoscopic excision of endometriosis. Her postoperative haemoglobin is 8.1 g/dL. Six hours postoperatively she complains of increased umbilical swelling, abdominal pain and shortness of breath and she appears pale. A repeat full blood count now shows a haemoglobin count of 6.5 g/dL. What are the most appropriate steps you should take next?

A. Transfuse one unit of cross-matched packed red cells and await events
B. Volume replacement with colloids and reassessment of the haemoglobin level
C. D-dimer and computed tomography (CT) pulmonary angiogram (CTPA)
D. Insertion of a large-bore nasogastric tube on free drainage
E. Transfuse four units of cross-matched packed red cells and return to
theatre for further laparoscopy

A

E. Transfuse four units of cross-matched packed red cells and return to
theatre for further laparoscopy

a dropping haemoglobin post-operatively is strongly suggestive of an ongoing bleed. The umbilical swelling, pain and anaemic symptoms add weight to this.

(A), (B), and (C) are essentialy conservative approaches and completely inapropriate here.

I don’t even know what (D) would solve, this is clearly not a bowel obstruction.

It’s got to be (E)

120
Q

A 54-year-old woman presents to her GP with a 1-year history of bloating, early
satiety and occasional crampy pelvic pain. She was diagnosed a year ago with
irritable bowel syndrome (IBS). A serum CA 125 is 62 IU/mL (normal range <36 IU/mL). What is the most appropriate management?

A. Pelvic examination and pipelle biopsy
B. Ultrasound of the abdomen and pelvis
C. Computed tomography of the abdomen and pelvis
D. Urgent referral to the gynaecology clinic under the 2-week rule for
suspected cancers
E. Trial of mebeverine and lifestyle modification

A

B. Ultrasound of the abdomen and pelvis

Given the histoy of abdominal bloating and a raised CA125 ovarian cancer is to be investigated here.

A pipelle biopsy (A) is used in endometrial cancer.

CT abdo/pelvis (C) is an assesment carried out for staging purposes if a diagnosis of ovarian malignancy is made.

At this stage we are not ready to refer (D) as we still need to carry out an assesment, which would be an ultrasound scan (B)

121
Q

A 24-year-old woman is admitted to the gynaecology ward with a 4-day history
of severe hyperemesis gravidarum. She has been unable to tolerate food or fluid
orally for 2 days. On the second day of admission she develops signs of a severe
pneumonia. This is presumed to be a hospital-acquired infection. She deteriorates
rapidly. An arterial blood gas shows:

pH 7.68
PO2 10.0 kPa
PCO2 4.26 kPa
HCO3 32 mmol/L
K+ 1.9 mmol/L
Lactate 1.2 mmol/L

What is the most accurate description of the acid-base disorder?
A. Metabolic alkalosis
B. Respiratory alkalosis
C. Mixed respiratory alkalosis and metabolic acidosis
D. Respiratory alkalosis with inadequate respiratory compensation
E. Mixed metabolic alkalosis and respiratory alkalosis

A

E. Mixed metabolic alkalosis and respiratory alkalosis

From the history of frequent vomiting we know that this woman is likely to have a metabolic alkolosis. We straight away see that her pH is severely alkolotic. But the next abnormality is a low CO2, which doesn’t fit with a pure metabolic alkolosis (A).

We see a hypokalemia also, this represent’s the metabolic component at work here and confirms our first thoughts from the history. She is losing anions in th eform of K+ and H+ in the vomitus.

We can confidently say this is a mixed alkolosis (E)

The respiratory alkolosis is likely due to tachypnea, due to the pneumonia.

122
Q

A 61-year-old woman has recently been diagnosed with a stage 1a endometrial
carcinoma. She has had four children, she has mild utero-vaginal prolapse and she has never been operated on. She needs to have surgery. You see her in clinic and talk about the different operations available to her. Which is the most appropriate operation?

A. Wertheim’s hysterectomy
B. Total abdominal hysterectomy
C. Laparoscopic hysterectomy
D. Subtotal hysterectomy
E. Posterior exenteration

A

C. Laparoscopic hysterectomy

Telling you that this woman has never had surgery does tell you that the question author is thinking of a laproscopic approach. In this case she has a mild prolapse already, hasn’t got adhesions die to surgery, and has an early stage endometrial cancer. The recovery will be speedier.

Of the other answers;

A Wetheim’s (A) is an option for cervical cancer. It removes the uterus and upper third of the vagina, which is not indicated here.

A total abdominal (B) would be appropriate if there was involvement of the ovaries also.

A subtotal hysterectomy (D) would preserve the cervix, this would not be appropriate for a curative operation for endometrial cancer.

Posterior exenteration (E) is a Wetheim’s and a bowel resection, it’s not indicated in early stage endometrial cancer.

123
Q

A 58-year-old woman presents to the clinic with post-menopausal bleeding. A
pipelle biopsy confirms adenocarcinoma of the endometrium. Further imaging of
the pelvis shows that there is spread of the tumour outside of the uterus into the
left adnexa. There is no other spread. What is the most likely stage of the tumour?

A. Stage 1A
B. Stage II

C. Stage IIIA
D. Stage IVA
E. Stage IIIC2

A

C. Stage IIIA

FIGO staging generally follows a similar pattern acroos different cancers. In the case of endometrial cancer;

  • Ia - confined to the uterus with less than half the myometrium invaded
  • Ib - greater than half the myometrium invaded
  • II - beyond the myometrium but not past the basement layer of the uterus
  • IIIa - adnexal or serosal involvement
  • IIIb - vaginal / parametrial involvement
  • IIIc1 - pelvic lymph nodes
  • IIIC2 - Para-aortic lymph nodes +- pelvic nodes
  • IVa - Bladder or bowel
  • IVb - distant mets
124
Q

A 65-year-old woman is referred by her GP to the gynaecology clinic with
increasing bloating and a raised CA 125 level. A CT scan shows an irregular,
enlarged left ovary and several well-circumscribed nodular lesions in the liver and
on the omentum which are highly suspicious for metastatic ovarian cancer. What
is the most appropriate treatment regimen?

A. Total hysterectomy, bilateral salpingo-oophorectomy and omentectomy
along with concomitant stereotactic radiotherapy of the liver lesions
B. Total hysterectomy, bilateral salpingo-oophorectomy, omentectomy,
aortopelvic lymphadenectomy
C. Staging laparotomy and optimal cytoreduction
D. Palliative care
E. Total pelvic exenteration

A

C. Staging laparotomy and optimal cytoreduction

Staging of ovarian cancer is primarily surgical (C), imaging doesn’t replace histological analysis. This is the preffered option in any patient fit for surgery, regardless of how advanced the disease.

Offering palliation in a fit patient is innapropriate (D) unless by patient choice.

Removing the aortic lymph nodes (B) has little evidence for improving survival and carries a large risk.

Concominant radiotherapy to a differnt organ (A) at the same time as radical surgery is a significant risk.

Total Exenteration (E) is the removal of the bladder, rectum and reproductive organs. It is ususally used for vulval, rectal, uterine, and vaginal cancers without distant spread. It is not appropriate here.

125
Q

A 62-year-old woman presents to accident and emergency with shortness of
breath. Examination reveals reduced breath sounds and a swollen, distended
abdomen. Chest x-ray demonstrates a left-sided pleural effusion. On further
questioning the woman has had a poor appetite for the last 6 months and recently
had some vaginal bleeding. An ultrasound revealed large quantities of ascites,
which were drained. Analysis of the ascites shows a high protein content. What is
the most likely diagnosis?

A. Congestive cardiac failure (CCF)
B. Carcinoma of the ovary
C. Meigs’ syndrome
D. Cirrhosis of the liver
E. Carcinoma of the cervix

A

B. Carcinoma of the ovary

The vaginal bleeding, bloating, and the poor appetite are allarming signs for an advanced gyne malignancy. The high protein content of the ascities points towards malignant or inflammatory processes., rather than CCF (A) or Cirhosis (D).

Meigs’ syndrome (C) is a pleural effussion, classically on the right, associated with a benign ovarian fibroma. It is a very rare syndrome and doesn’t explain the red flag symptoms here.

Of the two malignancies here, cervical cancer (E) tends to present earlier and this level of systemic involvement is less likely. Ovarian cancer (B) is more classically associated with massive abdominal distension.

126
Q

A 28-year-old woman attends accident and emergency unable to walk because
she is so faint. She has had heavy vaginal bleeding for 4 hours since she engaged
in sexual intercourse with a new partner, which she described as ‘rough and
very painful’. She is still bleeding and cannot tolerate vaginal examination
due to the pain. A point-of-care haemoglobin estimation is 6.4 g/dL and she is
haemodynamically unstable. What is the most appropriate management?

A. Discharge with oral iron supplementation and follow up in the
gynaecology clinic in 2 days
B. Discharge with oral iron supplementation and follow up on the ward in
24 hours
C. Admit, resuscitate and prepare her for immediate transfer to theatre
D. Admit to the gynaecology ward, cross-match four units of packed red
cells and send a formal full blood count
E. Admit to the gynaecology ward having packed the vagina

A

C. Admit, resuscitate and prepare her for immediate transfer to theatre

So this is a grim question…

Anyway…

The severe, and symptomatic, anaemia indicates this woman has lost a significant amount of blood. She is also actively bleeding, discharging her at this stage (A or B) would be extremely negligent.

You need to asses the damage to the genital area, she connot tolerate this so needs an examination under anesthesia (C)

If she cannot tolerate an examination, packing will be impossible (E)

And option (D) does nothing to rectify the active bleeding.

127
Q

A 64-year-old woman with asthma is admitted to the ward prior to an elective
vaginal hysterectomy for symptomatic uterine prolapse. Her medications
include Seretide (fluticasone/salmeterol 500/50) four times daily and oral
prednisolone 20 mg twice daily. What is the most important peri-operative
consideration?

A. Steroid cover with 50 mg hydrocortisone intravenously at induction of
anaesthesia
B. Steroid cover with 100 mg hydrocortisone intravenously at induction of
anaesthesia
C. Steroid cover with 50 mg hydrocortisone intravenously at induction of
anaesthesia and 50 mg 8-hourly for 3 days
D. Bronchodilator cover with intravenous salbutamol infusion
postoperatively
E. Continue regular medications and postoperative review by respiratory
physician

A

C. Steroid cover with 50 mg hydrocortisone intravenously at induction of
anaesthesia and 50 mg 8-hourly for 3 days

Patients taking more than 10mg of steroids a day for more than 3 months will almost certainly have a blunted Hypothalamic-pituitary access. This means that they cannot ount an appropriate physiological increase in corticosteroid levels after surgery. They will require supportive steroid therapy. Option (C) is the standard regime for major gyne surgery, any single dose of steroid (A or B) is not going to be sufficient cover until she is eating and drinking again and can resume oral steroids.

option (D) represents a covering treatment against asthma attacks and would not be routinely used.

Finally, any patient with well controlled asthma will not need a respiratory review (E)

128
Q

A woman is undergoing surgery to enhance the cosmetic appearance of her labia.
A bleeding vessel is encountered at the labia majora which cannot be controlled
through pressure alone. The surgeon believes it to be a branch of the posterior
labial artery. The posterior labial artery is a branch of which artery?

A. Internal pudendal artery
B. Inferior gluteal artery
C. Uterine artery
D. Obturator artery
E. Inferior vesical artery

A

A. Internal pudendal artery

which itself arises from the internal iliac

B. Inferior gluteal artery - Supplies the buttock and posterior thigh
C. Uterine artery - Supplies the uterus
D. Obturator artery - Supplies the medial compartment of the thigh
E. Inferior vesical artery - supplies the lower part of thebladder

129
Q

Following surgery to place a tension-free transobturator tape for stress incontinence,
a 54-year-old woman loses some sensation in part of her labia anterior to the anus.
Damage has most likely been caused to which nerve?

A. Perineal nerve
B. Peroneal nerve
C. Pudendal nerve
D. Dorsal nerve of clitoris
E. Inferior anal nerve

A

A. Perineal nerve

The perineal nerve is a branch of the pudendal nerve, it provides sensory enervation to parts of the labia and perineum.

Compromise of the pudendal nerve (C) would cause much more severe deficits.

The Peroneal nerve (B) also known as the common fibular nerve, supplies the anterior and lateral aspects of the lower leg.

The remaining two answers are also branches of the pudendal nerve. The Dorsal nerve of clitoris (D) supplies part of the clitoris, shock horror…

The inferior anal nerve (E) supplies the anal skin and external anal sphincter.

130
Q

A 54-year-old woman with a history of significant ischaemic heart disease
undergoes vaginal hysterectomy for symptomatic uterine prolapse. She develops
significant surgical site bleeding which is repaired at reoperation the same day. Her postoperative haemoglobin is 6.4 g/dL. Later the same day she develops chest pain. Her observations, blood gas and cardiac enzymes are within normal limits. An electrocardiogram (ECG) shows sinus rhythm without ST changes. She is charted for thromboprophylaxis. What is the most likely cause of the chest pain?

A. Non ST-elevation myocardial infarction
B. Anaemia
C. Pulmonary embolism
D. Atelectasis
E. Postoperative sepsis

A

B. Anaemia

This woman’s history of ischemic heart disease means that this level of anaemia (B) is causing a degree of sub-optimal perfusion of the cardiac muscle. This is causing the cheast pain, but isn’t actually infarcting (A). The norml troponic and normal ECG confirm ths is not a MI.

This woman is on thromboprophylaxis, this makes a PE an unlikely diagnosis (C)

Without fever and so rapidly after the operation sepsis (E) is unlikely.

The patient dosen’t report any SOB or cough, combined with teh parid onset after surgery this makes Atelectasis (D) very unlikely.

131
Q

A 46-year-old woman is returned to the ward from the recovery room following a
routine vaginal hysterectomy for heavy periods and prolapse. The estimated blood
loss at operation was 200 mL. Two hours later the ward sister becomes concerned that her urine output is low and calls the doctor. Her observations show: pulse 115 bpm, BP 90/62 mmHg, temperature 37.1°C. What are the most appropriate next steps in her management?

A. Aggressive fluid resuscitation, alert the operating surgeon and prepare
for a return to theatre
B. Fluid challenge, haemoglobin estimation and arterial blood gas
C. Vaginal examination, haemoglobin estimation and arterial blood gas
D. Establish large-bore intravenous access, alert the operating surgeon and
perform arterial blood gas
E. Establish large-bore intravenous access, alert the operating surgeon and
perform a fluid challenge

A

A. Aggressive fluid resuscitation, alert the operating surgeon and prepare
for a return to theatre

This level of systemic shock in a fit, young patient is seriously alarming. It needs to be assumed to be a haemorrhage until proven otherwise.

Although the idea of establishing large-bore IV access is always a good idea in a patient such as this, neither a ABG (D) or a fluid challenge (E) are definitive steps in this woman’s management. This woman needs decisive life saving resuscitation and potentialy another surgery, options (B) and (C) are simply not enough for a potentially exsanguinating patient.

Fluid resuscitation needs to be aggressive and she needs to be back in theatre where the team can decide on next steps (A). In a case such as this you should always tell the origional surgeon that thier patient is showing signs of a significant post-operative bleed.

132
Q

The peritoneal lining drapes over the pelvic viscera and forms the part of the
peritoneal cavity. Which is the most inferior extent of the peritoneal cavity?

A. Vesicouterine pouch
B. Paravesical fossa
C. Rectouterine pouch (Pouch of Douglas)
D. Pararectal fossa
E. Rectovesical pouch

A

C. Rectouterine pouch (Pouch of Douglas)

The pouch of Douglas (C) is the deepest extent of the peritoneal cavity, if you don’t remember this anatomical fact, just remember that this is the site that free fluid inevitibly ends up. It lies between the posterior uterus and the rectum.

The Vesicouterine pouch (A), unsurprisingly, lies between the bladder and anterior uterus.

Paravesical fossa (B) are either side of the peritoneal covered bladder.

Pararectal fossa (D) is similarly either side of teh uper rectum

The Rectovesical pouch (E) doesn’t exist in the female pelvis as this is where the uterus lies.

133
Q

A 74-year-old woman has an annual health check up with her private insurer.
They arrange an ultrasound scan that shows a cyst on her right ovary. It is
multiloculated and has solid components. She is post-menopausal and otherwise
well. A doctor has sent for a CA 125 which comes back as 120 U/mL. What is her
risk of malignancy index score (RMI)?

A. 120
B. 240
C. 60
D. 720
E. 480

A

D. 720

The RMI uses several factors in it’s calculation;

  • number of pathological ultrasound features (out of 5)
  • CA 125 score
  • Menopause status (score 3 for post-menopausal, 0 otherwise)

Ultrasound features are;

  1. Bilateral cysts
  2. multiloculated cysts
  3. Cysts with solid components
  4. ascites
  5. metastases

Thus RMI = (U/S score) x (CA 125) x 3 (if postmenopausal)

in this case; 2x120x3 = 720

This means she has a 75% chance of cancer, she should be reffered for investigation and work-up

134
Q

A 21 year old comes to the clinic with a history of intermenstrual bleeding for the
last 6 weeks. She has regular periods and does not experience post-coital bleeding.
She is not on the oral contraceptive pill and has no other past medical history.
What is the most appropriate first line investigation?

A. Hysteroscopy and biopsy
B. Cervical smear test
C. Triple swabs for pelvic infection
D. Ultrasound scan of the pelvis
E. Pipelle biopsy

A

C. Triple swabs for pelvic infection

In a 21 year old patient with new onset inter-menstrual bleeding, pelvic infection needs to be investigated with a triple swab (C)

She has yet to start the national smear test programme so a smear culd be offered (B) but it isn’t the first investigation required.

The biosy options (A and E) are rather extreme choices where there is no strong evidence of malignancy, especially in a very young woman.

If the first line investigations didn’t provide answers,t ehn an ultrasound scan (D)looking at endometrial thickness would be warranted. Only if this was abnormal would you choose to do a pipelle biopsy (E) of the endometrium.

135
Q

Two days after undergoing posterior exenteration for recurrence of cervical
adenocarcinoma a 53-year-old woman develops a tachypnoea, tachycardia of
125 bpm and a fever of 39°C. Blood cultures have grown methicillin-resistant
Staphylococcus aureus (MRSA). She requires intravenous vasopressors. What is the most appropriate diagnosis?

A. Sepsis
B. Systemic inflammatory response syndrome
C. Septic shock
D. Septicaemia
E. Adult respiratory distress syndrome

A

C. Septic shock

We can think about (B), (A) and (C) as a spectrum of conditions.

SIRS (B) is when the body mounts a continuing systemic inflammatory response to a pathological insult. It is charecterised by a temprature of <36c or >38c, HR >90bpm, tachypnoe >20 minutes, or white cells <4 or >12. The patient meets these initial criteria.

If, however, there is evidence of a pthogen then SIRS is classed as sepsis (A).

If that sepsis leads to haemodynamic compromise and shock (as we see here with the use of vasopressors), then it is termed septic Shock (C). This is our answer in this scenario.

Scepticaemia (D) is simply the isolation of a pthogen from the bloodstream

Adult respiratory distress syndrome (E), is caused by insult to the lungs leading to impaired gas exchange. There are also systemic effects in this condition, but it is not the scenarion before us.

136
Q

A 60-year-old woman is undergoing abdominal hysterectomy for a fibroid uterus.
During suture ligation of the right uterine pedicle, iatrogenic injury to the ureter
is confirmed. Which of the following statements is correct?

A. The ureter passes through the mesometrium and posterior to the uterine
artery on its course to the urinary bladder
B. The ureter passes outside of the mesometrium and anterior to the
uterine artery on its course to the urinary bladder

C. The ureter lies posterior to the internal iliac artery and lateral to the
obturator nerve opposite the lower part of the greater sciatic notch
D. The ureter passes inferior to the cardinal ligament before coursing
anteriorally to enter the urinary bladder
E. The ureter is not closely related to the uterine arteries

A

A. The ureter passes through the mesometrium and posterior to the uterine
artery on its course to the urinary bladder

The
ureters run down the lateral pelvic side walls, along the anterior border
of the greater sciatic notch and under the peritoneum. Medial to the
obturator nerve, the ureter lies anterior to the internal iliac artery. It enters
the pelvis crossing over the iliac vessels at the level of the bifurcation.
The ureter passes underneath the uterine artery about 15 mm lateral to the
supravaginal cervix (remember ‘water under the bridge’) before coursing
towards the urinary bladder.

137
Q

A 57-year-old woman has been referred by her GP under the 2-week suspected
cancer referral approach with vaginal bleeding. She has been post-menopausal
for the last 4 years and she has been taking Elleste Duet to treat her vasomotor
symptoms. Two weeks ago, after reading about the risks associated with hormone
replacement therapy (HRT) she stopped taking any medication. This is the first
unscheduled bleeding she has ever had. She had a normal smear 2 years ago and
is otherwise well. What would be your first line investigation?

A. Pipelle biopsy
B. Hysteroscopy
C. Smear test
D. Ultrasound of the pelvis
E. CT abdomen and pelvis

A

D. Ultrasound of the pelvis

Although post-menopausal bleeding can be a symptom of ovarian, endometrial and cervical caner, in this case it is more likely to be as a result of her stopping HRT.

We shouldn’t rely on this assumption though and should investigate.

A Hysteroscopy (B) and a pipelle biopsy (A) are both invasive procedures, they are not warrented before the first line investigation of an ultrasound scan to check the endometrial thickness (D)

Her smear tests are up to date (C), so this is unecessary.

Finally a CT abdo pelvis (E) is a staging tool rather than a diagnostic tool. It’s not first line.

138
Q

A 39-year-old woman attends the gynaecology clinic complaining of long-standing
pelvic pain. Routine bimanual examination and abdominal ultrasonography do
not detect any abnormality. At diagnostic laparoscopy, multiple tiny dark brown
nodular lesions are noted covering the surface of the uterus, tubes and left ovary,
as well as in the Pouch of Douglas. Which finding is most likely from histological
examination of the excised lesions?

A. Krukenberg tumour
B. Vacuolated clear cells
C. Endometrial glands with stromal cells
D. Multiple leiomyomata
E. Enucleolated hyperplastic smooth muscle cells

A

C. Endometrial glands with stromal cells

The pelvic examination findings are consistent with endometriosis, particularly of note is the description of tiny brown nodules covering the majority of the pelvic structures. It follows that any histology of the excised nodules will show endometrial architecture (C).

A krukenberg tumour (A) is a metastatic tumour of the ovary from a gastric primary.

Vacuolated clear cells (B) are seen in clear cell adenocarcinoma

Both (E) and (D) are suggestive of fibroids.

139
Q

A 21-year-old woman with dysmenorrhorea, dyspareunia and dyschezia has been
scheduled for a laparoscopy to investigate possible endometriosis. You are asking
for her consent and you describe the risks of laparoscopy, which include bleeding
and damage to blood vessels, viscera and nerves. Which of the following is not at
risk when inserting a lateral port?

A. Superficial epigastric artery
B. -
C. Iliohypogastric nerve
D. Superior epigastric artery
E. Ilioinguinal nerve

A

D. Superior epigastric artery

-unfortunately another badly written question-

During port insertion any structure, deep or superficial is at risk of damage. Extreme care must be taken.

The superior epigastric artery (D) runs paralell to the mid line and anastomeses with the superficial epigastric artery (A) near the umbilicus. It can be seem that the superficial is at risk as it arises from the Femoral and travels to the umbilicus, the superior is nowhere near the site of lateral port insertion. .

The two nerves here (C and E) both arises from L1 and are at risk of damage.

Option (B) origionally said External iliac vein, this becomes the common iliac at the inguinal ligamnet so is nowhere near the site of port insertion.